Разное

Статистика стандартное отклонение: Стандарное отклонение

Содержание

Дисперсия, среднеквадратичное (стандартное) отклонение, коэффициент вариации в Excel

Из предыдущей статьи мы узнали о таких показателях, как размах вариации, межквартильный размах и среднее линейное отклонение. В этой статье изучим дисперсию, среднеквадратичное отклонение и коэффициент вариации.

Дисперсия

Дисперсия случайной величины – это один из основных показателей в статистике. Он отражает меру разброса данных вокруг средней арифметической.

Сейчас небольшой экскурс в теорию вероятностей, которая лежит в основе математической статистики. Как и матожидание, дисперсия является важной характеристикой случайной величины. Если матожидание отражает центр случайной величины, то дисперсия дает характеристику разброса данных вокруг центра.

Формула дисперсии в теории вероятностей имеет вид:

То есть дисперсия — это математическое ожидание отклонений от математического ожидания.

На практике при анализе выборок математическое ожидание, как правило, не известно. Поэтому вместо него используют оценку – среднее арифметическое. Расчет дисперсии производят по формуле:

где

s2 – выборочная дисперсия, рассчитанная по данным наблюдений,

X – отдельные значения,

– среднее арифметическое по выборке.

Стоит отметить, что у такого расчета дисперсии есть недостаток – она получается смещенной, т.е. ее математическое ожидание не равно истинному значению дисперсии. Подробней об этом здесь. Однако при увеличении объема выборки она все-таки приближается к своему теоретическому аналогу, т.е. является асимптотически не смещенной.

Простыми словами дисперсия – это средний квадрат отклонений. То есть вначале рассчитывается среднее значение, затем берется разница между каждым исходным и средним значением, возводится в квадрат, складывается и затем делится на количество значений в данной совокупности. Разница между отдельным значением и средней отражает меру отклонения. В квадрат возводится для того, чтобы все отклонения стали исключительно положительными числами и чтобы избежать взаимоуничтожения положительных и отрицательных отклонений при их суммировании. Затем, имея квадраты отклонений, просто рассчитываем среднюю арифметическую. Средний – квадрат – отклонений. Отклонения возводятся в квадрат, и считается средняя. Теперь вы знаете, как найти дисперсию.

Расчет дисперсии в Excel

Генеральную и выборочную дисперсии легко рассчитать в Excel. Есть специальные функции: ДИСП.Г и ДИСП.В соответственно.

В чистом виде дисперсия не используется. Это вспомогательный показатель, который нужен в других расчетах. Например, в проверке статистических гипотез или расчете коэффициентов корреляции. Отсюда неплохо бы знать математические свойства дисперсии.

Свойства дисперсии

Свойство 1. Дисперсия постоянной величины A равна 0 (нулю).

D(A) = 0

Свойство 2. Если случайную величину умножить на постоянную А, то дисперсия этой случайной величины увеличится в А2 раз. Другими словами, постоянный множитель можно вынести за знак дисперсии, возведя его в квадрат.

D(AX) = А2 D(X)

Свойство 3. Если к случайной величине добавить (или отнять) постоянную А, то дисперсия останется неизменной.

D(A + X) = D(X)

Свойство 4. Если случайные величины X и Y независимы, то дисперсия их суммы равна сумме их дисперсий.

D(X+Y) = D(X) + D(Y)

Свойство 5. Если случайные величины X и Y независимы, то дисперсия их разницы также равна сумме дисперсий.

D(X-Y) = D(X) + D(Y)

Среднеквадратичное (стандартное) отклонение

Если из дисперсии извлечь квадратный корень, получится среднеквадратичное (стандартное) отклонение (сокращенно СКО). Встречается название среднее квадратичное отклонение и сигма (от названия греческой буквы). Общая формула стандартного отклонения в математике следующая:

На практике формула стандартного отклонения следующая:

 

Как и с дисперсией, есть и немного другой вариант расчета. Но с ростом выборки разница исчезает.

Расчет cреднеквадратичного (стандартного) отклонения в Excel

Для расчета стандартного отклонения достаточно из дисперсии извлечь квадратный корень. Но в Excel есть и готовые функции: СТАНДОТКЛОН.Г и СТАНДОТКЛОН.В (по генеральной и выборочной совокупности соответственно).

Среднеквадратичное отклонение имеет те же единицы измерения, что и анализируемый показатель, поэтому является сопоставимым с исходными данными.

Коэффициент вариации

Значение стандартного отклонения зависит от масштаба самих данных, что не позволяет сравнивать вариабельность разных выборках. Чтобы устранить влияние масштаба, необходимо рассчитать коэффициент вариации по формуле:

По нему можно сравнивать однородность явлений даже с разным масштабом данных. В статистике принято, что, если значение коэффициента вариации менее 33%, то совокупность считается однородной, если больше 33%, то – неоднородной. В реальности, если коэффициент вариации превышает 33%, то специально ничего делать по этому поводу не нужно. Это информация для общего представления. В общем коэффициент вариации используют для оценки относительного разброса данных в выборке.

Расчет коэффициента вариации в Excel

Расчет коэффициента вариации в Excel также производится делением стандартного отклонения на среднее арифметическое:

=СТАНДОТКЛОН.В()/СРЗНАЧ()

Коэффициент вариации обычно выражается в процентах, поэтому ячейке с формулой можно присвоить процентный формат:

Коэффициент осцилляции

Еще один показатель разброса данных на сегодня – коэффициент осцилляции. Это соотношение размаха вариации (разницы между максимальным и минимальным значением) к средней. Готовой формулы Excel нет, поэтому придется скомпоновать три функции: МАКС, МИН, СРЗНАЧ.

Коэффициент осцилляции показывает степень размаха вариации относительно средней, что также можно использовать для сравнения различных наборов данных.

Таким образом, в статистическом анализе существует система показателей, отражающих разброс или однородность данных. 

Ниже видео о том, как посчитать коэффициент вариации, дисперсию, стандартное (среднеквадратичное) отклонение и другие показатели вариации в Excel.

Поделиться в социальных сетях:

Отклонение (статистика) — Deviation (statistics)

В математике и статистике , отклонение является мерой разности между наблюдаемым значением переменного и другим значением, часто этим переменное это средний . Знак отклонения сообщает направление этой разности (отклонение положительное , когда наблюдаемое значение превышает опорное значение). Величина значения указывает размер разницы.

Типы

Отклонение, которое представляет собой разницу между наблюдаемым значением и истинным значением интересующей величины (где истинное значение обозначает ожидаемое значение, например среднее значение генеральной совокупности), является ошибкой.

Отклонение, которое представляет собой разницу между наблюдаемым значением и оценкой истинного значения (например, среднее значение выборки; ожидаемое значение выборки может использоваться как оценка ожидаемого значения совокупности), является остатком . Эти концепции применимы для данных в то интервал и соотношения уровней измерения.

Беззнаковое или абсолютное отклонение

В статистике , то абсолютное отклонение какого — либо элемента в наборе данных является абсолютная разница между этим элементом и в данной точке. Обычно отклонение рассчитывается от центрального значения и трактуется как некоторый тип среднего , чаще всего медианы, а иногда и среднего значения набора данных:

Dязнак равно|Икся-м(Икс)|,{\ displaystyle D_ {i} = | x_ {i} -m (X) |,}

где

D i — абсолютное отклонение,
x i — элемент данных,
m ( X ) — выбранная мера центральной тенденции набора данных — иногда среднее ( ), но чаще всего медиана .Икс¯{\ displaystyle {\ overline {x}}}

Меры

Среднее знаковое отклонение

Для несмещенной оценки среднее значение отклонений со знаком по всему набору всех наблюдений от значения параметра ненаблюдаемой совокупности в среднем равно нулю для произвольно большого количества выборок. Однако по построению среднее знаковых отклонений значений от выборочного среднего значения всегда равно нулю, хотя среднее знаковое отклонение от другого показателя центральной тенденции, такого как медиана выборки, не обязательно равно нулю.

Дисперсия

Статистика распределения отклонений используется в качестве меры статистической дисперсии .

Нормализация

Отклонения имеют единицы измерения шкалы (например, метры при измерении длин). Безразмерность можно сделать двумя способами.

Один из способов — это деление на меру масштаба ( статистическую дисперсию ), чаще всего стандартное отклонение генеральной совокупности при стандартизации или стандартное отклонение выборки при студентизации (например, стьюдентизированный остаток ).

Можно масштабировать вместо этого места, а не дисперсию: формула для отклонения процентов является наблюдаемое значение минус принятое значение , деленное на общепринятом значении , умноженное на 100%.

Смотрите также

<img src=»https://en.wikipedia.org/wiki/Special:CentralAutoLogin/start?type=1×1″ alt=»» title=»»>

Первичные статистики в психологическом исследовании: среднее, стандартное отклонение

Первичные описательные статистики – это наиболее простые характеристики, которыми можно описать психологические данные, которые были получены в ходже тестирования испытуемых.

К наиболее часто используемым в курсовых и дипломных по психологии описательным статистикам можно отнести:

  • среднее значение;
  • стандартное отклонение.

 

Среднее значение

Простейшая математическая процедура, которую необходимо освоить студенту-психологу при написании диплома – расчет среднего значения.

Среднее значение или среднее арифметическое – это число, получаемое как сумма нескольких показателей, деланная на количество этих показателей. Например, в результате тестирования были получены показатели тревожности в группе из 10-ти человек. Чтобы получить среднее значение тревожности по группе нужно сложить показатели всех испытуемых, а затем получившуюся сумму разделить на 10.

Среднее значение характеризует группу целиком. Зная среднее можно оценить показатели каждого испытуемого относительно остальных. Например, измеряемая в приведённом выше примере тревожность могла быть от 1 до 5 баллов. Пусть средняя по группе тревожность оказалась 3,5 балла. Тогда, показатель испытуемого в 4 балла можно считать относительно высоким, а в 2 балла- относительно низким.

Среднее значение относится к показателям центральной тенденции и отражает степень выраженности показателя в группе. Стандартное отклонение отражает степень изменчивости признака в группе, но о нем речь впереди.

Среднее значение какого-либо показателя характеризует группу в целом и позволяет сравнивать ее с другими группами. Например, проведена диагностика уровня эмпатии в группе мужчин и женщин. Как узнать, влияет ли пол на способность к эмпатии. Один из способов – найти средний уровень этого показателя в группах мужчин и женщин. Например, в группе женщин средний уровень эмпатии равен 23,5 баллов, а в группе мужчин – 17,7 баллов. Как видно, в среднем у женщин эмпатия выше, чем у мужчин.

Важно отметить, среднее значение – это не просто число, а – статистическое – полученное в результате особой процедуры. Поэтому и сравнивать средние значения как обычные числа нельзя. Для сравнения средних значений используются дополнительные процедуры – расчет статистических критериев. Например, U-критерий Манна-Уитни или t-критерий Стъюдента.

Среднее – это не единственный статистический показатель, который отражает выраженность переменной в группе. Аналогичную функцию выполняют мода и медиана. Однако они редко используются в дипломах по психологии.

Средние значения выраженности психологических показателей в курсовой или дипломной по психологии представляются в виде таблиц и диаграмм. В таблицах среднее обозначается буквой «М».

 

Стандартное отклонение

Если среднее арифметическое отражает выраженность показателя в группе, то стандартное отклонение (среднеквадратичное отклонение) показывает его разброс данных или изменчивость. Чем больше величина стандартного отклонения, тем больше разброс показателей в группе испытуемых.

Например, группу мальчиков протестировали методикой на выявление уровня эгоцентризма, показатели которого изменяются от 1 до 10. Расчет среднего показал М=6,5, а стандартное отклонение σ=3 (стандартное отклонение обозначается буквой «сигма»). Эти данные позволяют нам говорить о том, что подавляющее большинство показателей эгоцентризма мальчиков укладываются в диапазон от 3,5 до 9,5 (среднее плюс/минус стандартное отклонение – М ± σ).

Если при тестировании группы девочек среднее значение М=5, а стандартное отклонение σ=1, то большинство испытуемых этой группы имеют эгоцентризм в диапазоне от 4 до 6 (5 ± 1).

Анализирую такие данные в дипломе по психологии можно указать, что средний уровень эгоцентризма у мальчиков больше, чем у девочек. При этом разброс показателей эгоцентризма у мальчиков также больше, чем у девочек, то есть, в группе мальчиков есть испытуемые с очень низкими и очень высокими показателями относительно среднего. У девочек показатели менее «разбросаны» относительно среднего.

 

Расчет среднего и стандартного отклонения

Формула расчета среднего очень проста и этот параметр можно рассчитать вручную.

Пример расчёта среднего

В таблице приведены показатели, полученные по тесту диагностики уровня одиночества у 64-х испытуемых.

№ исп.

Уровень одиночества

1

13

2

14

3

5

4

11

5

17

6

9

7

18

8

6

9

9

10

9

11

15

12

14

13

7

14

9

15

8

16

13

17

12

18

14

19

19

20

15

21

11

22

15

23

6

24

8

25

8

26

8

27

5

28

20

29

5

30

9

31

7

32

7

33

11

34

15

35

7

36

7

37

9

38

8

39

11

40

17

41

10

42

18

43

15

44

14

45

15

46

4

47

8

48

15

49

17

50

14

51

4

52

8

53

18

54

14

55

14

56

9

57

1

58

7

59

11

60

4

61

14

62

11

63

6

64

17

Найдем средний уровень переживания одиночества в группе.

М=(13 + 14+ 5+ 11+ 17+ 9+ 18+ 6+ 9+ 15+ 14+ 7+ 9+ 8+ 13+ 12+ 14+ 19+ 15+ 11+ 15+ 6+ 8+ 8+ 8+ 5+ 20+ 5+ 9+ 7+ 7+ 11+ 15+ 7+ 7+ 9+ 8+ 11+ 17+ 10+ 18+ 15+ 14+ 15+ 4+8+15+17+14+4+8+18+14+14+9+1+7+11+4+14+11+6+17)  / 64=10,92

Как видим, если испытуемых достаточно много, то рассчитывать среднее вручную задача трудоемкая.

Еще более трудоемкий процесс — расчёт стандартного отклонения. Не буду утомлять вас формулами, скажу лишь, что расчёт этого показателя сводится к тому, что суммируются квадраты разности показателей со средним значением. Затем эта сумма делится на число показателей и из полученного числа извлекается квадратный корень. Вручную такие вычисления делать хлопотно, и не нужно.

Чаще всего расчеты среднего и стандартного отклонения можно делать в статистических программах STATISTICA, SPSS и электронных таблицах Exсel.


Надеюсь, эта статья поможет вам написать работу по психологии самостоятельно. Если понадобится помощь, обращайтесь (все виды работ по психологии; статистические расчеты). Заказать

Среднее отклонение, стандартное отклонение и дисперсия в обработке сигналов

Добавлено 27 июля 2020 в 08:05

Сохранить или поделиться

В данной статье рассматриваются три описательных статистических меры с точки зрения приложений обработки сигналов.

В предыдущей статье, посвященной описательной статистике для инженеров-электронщиков, мы увидели, что центральную тенденцию набора данных могут передавать как среднее арифметическое, так и медиана. Несмотря на то, что медиана менее чувствительна к выбросам, в электронике и цифровой обработке сигналов чаще используется среднее арифметическое. Среднее арифметическое, по сути, является основным статистическим методом в электротехнике.

Однако для адекватного описания или понимания набора данных нам часто требуется нечто большее, чем только среднее арифметическое.

Когда мы сообщаем только о центральной тенденции, мы не учитываем важный аспект данных, а именно то, каким образом значения отклоняются от центральной тенденции.

Отклонение от среднего значения

Давайте представим, что мы оцифровали два аналоговых входных сигнала. Если мы преобразуем цифровые коды обратно в единицы вольт и построим графики по времени, они будут выглядеть следующим образом:

Рисунок 1 – График измеренных сигналов

Мы можем довольно хорошо угадать средние значения, просто взглянув на график: центральная тенденция синего сигнала равна 1,2 В, а красного сигнала – 0,8 В. Но если мы сообщим только о средних значениях, мы создадим впечатление, что единственное важное различие между этими двумя сигналами – это разница средних значений 0,4 В (или мы можем назвать это уровнем постоянной составляющей или смещением по постоянному напряжению). Очевидно, что это еще не всё.

Инженер-электронщик интуитивно идентифицирует эти сигналы как устойчивые сигналы постоянного напряжения (возможно, напряжения питания), которые содержат довольно много шума.

Что еще более важно, мы немедленно признаем, что синий сигнал значительно более шумный, чем красный сигнал. Это основное различие в шумовых характеристиках теряется, если рассматривать только среднее значение.

Кстати, почему мы замечаем шум в этих сигналах? Так как

  • отдельные значения заметно отклоняются от среднего значения,
  • они делают это так, что кажутся случайными, и
  • отклонения малы относительно среднего значения.

Когда статистик видит небольшие случайные отклонения от среднего значения, инженер-электронщик видит шум.

Среднее отклонение

Насколько шумные эти сигналы? Довольно шумные? Очень шумные? Попробуем дать более точный ответ на этот вопрос. Другими словами, нам нужно количественно определить отклонение в этих наборах данных.

Моя первая мысль при измерении отклонения состоит в том, чтобы найти расстояние между каждой точкой данных и средним значением, а затем вычислить среднее значение всех этих расстояний. Это даст нам среднее отклонение (также называемое средним абсолютным отклонением, MAD, mean absolute deviation), то есть типовое значение, на которое значения отклоняются от центральной тенденции.{N-1}|x[k]-\mu|\]

где N – количество значений в наборе данных, μ – среднее значение, а x[k] – сигнал, представленный как функция дискретной по времени переменной k.

Рисунок 2 – На этом графике горизонтальные линии показывают уровни напряжения, которые на величину одного среднего отклонения выше и ниже среднего значения.

Хотя среднее отклонение интуитивно понятно, оно не является самым распространенным методом количественной оценки склонности сигнала отклоняться от среднего значения. Для этого нам нужно стандартное отклонение.

Дисперсия и стандартное отклонение

В области электротехники проблема со средним отклонением состоит в том, что мы усредняем разности напряжений (или токов), и, следовательно, работаем в области амплитуд. Природа шумовых явлений такова, что при анализе шума мы делаем упор на мощности, а не на амплитуды, и, следовательно, нам нужен статистический метод, который работает в области мощностей.

К счастью, это просто.2}\]

Эта процедура генерирует статистическую меру, известную как стандартное отклонение, то есть усредненную мощность случайных отклонений сигнала, выраженную в виде амплитуды. Таким образом, если мы анализируем сигнал напряжения, стандартное отклонение имеет единицы измерения В, несмотря на то, что мы вычислили стандартное отклонение, используя квадрат отклонений напряжения.

Рисунок 3 – На этом графике горизонтальные линии показывают уровни напряжения, которые на величину одного стандартного отклонения выше и ниже среднего значения.

Дисперсия и стандартное отклонение по-разному выражают одну и ту же информацию. Хотя дисперсия, насколько я понимаю, более удобна в определенных аналитических ситуациях, стандартное отклонение обычно предпочтительнее, поскольку это число, которое можно непосредственно интерпретировать, как меру склонности сигнала отклоняться от среднего значения.

Заключение

Стандартное отклонение и дисперсия являются важными статистическими методами, которые часто фигурируют в технических и общественных науках. Я надеюсь, что данная статья помогла вам понять основную связь между этими понятиями и электрическими сигналами, и в следующей статье мы рассмотрим некоторые интересные подробности, связанные со стандартным отклонением.

Оригинал статьи:

Теги

ДисперсияОписательная статистикаСреднее отклонение / Среднее абсолютное отклонение / MAD (mean absolute deviation)Стандартное отклонениеСтатистикаЭлектротехника

Сохранить или поделиться

8 базовых понятий статистики для науки о данных | by Victoria Likhanova | NOP::Nuances of Programming

Статистика — это разновидность математического анализа, использующая количественные модели и репрезентации для анализа экспериментальных или реальных данных. Главное преимущество статистики — простота представления информации. Недавно я пересматривала материалы по статистике и выделила 8 основных понятий, которые должен знать каждый специалист по обработке данных:

  • дескриптивная аналитика;
  • вероятность;
  • среднее значение;
  • изменчивость;
  • взаимозависимость переменных;
  • вероятностное распределение;
  • проверка гипотезы и статистическая значимость;
  • регрессия.

Дескриптивная аналитика

Дескриптивная аналитика описывает события в прошлом и помогает бизнесу оценить эффективность деятельности, предоставляя всем участникам процесса контекст, необходимый для интерпретации информации.

Вероятность

Вероятность — это мера возможности наступления события при случайном эксперименте.

Дополнение: P(A)+P(A’) =1

Пересечение: P(A∩B)=P(A)P(B)

Объединение: P(A∪B)=P(A)+P(B)−P(A∩B)

Пересечение и объединение

Условная вероятность: P(A|B) — это мера возможности наступления одного события по отношению к другому/-им событию/-ям. P(A|B)=P(A∩B)/P(B), когда P(B)>0.

Независимые события: два события считаются независимыми, если наступление одного из них не влияет на возможность наступления другого. P(A∩B)=P(A)P(B), где P(A) != 0 и P(B) != 0 , P(A|B)=P(A), P(B|A)=P(B).

Взаимоисключающие события: два события считаются взаимоисключающими, если оба они не могут произойти в одно и то же время. P(A∩B)=0 и P(A∪B)=P(A)+P(B).

Теорема Байеса описывает вероятность наступления события, исходя из ранее известной информации об условиях, которые могут иметь отношение к этому событию.

Теорема Байеса (слева — апостериорная вероятность, справа — априорная вероятность)

Среднее значение

Среднее арифметическое: среднее значение набора данных.

Медиана: срединное значение упорядоченного набора данных.

Мода: наиболее часто встречающееся значение в наборе данных. Если таких значений несколько, это называется мультимодальным распределением.

Асимметрия: мера симметричности.

Эксцесс: мера, показывающая медленное или быстрое убывание “хвоста” данных относительно нормального распределения.

Асимметрия

Positive Skew — Положительная асимметрия
Symmetrical Distribution — Симметричное распределение
Negative Skew — Отрицательная асимметрия
Mode — Мода
Median — Медиана
Mean — Среднее арифметическое

Эксцесс

Kurtosis - Эксцесс
Frequency - Частота
Height - Высота
Leptokurtic - Положительный эксцесс
Mesokurtic - Нормальный эксцесс
Platykurtic - Отрицательный эксцесс

Изменчивость

Амплитуда: разница между минимальным и максимальным значениями в наборе данных.

Межквартильный размах (IQR): IQR = Q3−Q1

Межквартильный размах

Min - Минимум
Max - Максимум
Median - Медиана

Дисперсия: среднеквадратичное отклонение значений от среднего арифметического, показывающее разброс данных относительно него.

Стандартное отклонение: стандартный разброс между каждым отдельным значением и средним арифметическим, квадратный корень из дисперсии.

Выборочная и генеральная дисперсии и стандартное отклонение

Sample Variance - Выборочная дисперсия
Population Variance - Генеральная дисперсия
Sample Standard Deviation - Выборочное стандартное отклонение
Population Standard Deviation - Генеральное стандартное отклонение

Среднеквадратическая ошибка (SE): приблизительная величина стандартного отклонения выборочного распределения.

Описательная статистика для числовых переменных

6.2. Вывод статистических характеристик

Чтобы получить описательную статистику числовых переменных, можно щелкнуть в диалоге Frequencies на кнопке Statistics… (Статистика).
Откроется диалоговое окно Frequencies: Statistics (Частоты: Статистика).

Рис. 6.2: Диалоговое окно frequencies: Statistics

В группе Percentile Values (Значения процентилей) можно выбрать следующие варианты:

  • Quartiles (Квартили): Будут показаны первый, второй и третий квартили.
    Первый квартиль (Q1) — это точка на шкале измеренных значений, ниже (левее) которой располагаются 25% измеренных значений.
    Второй квартиль (Q2) — это точка, ниже которой располагаются 50% измеренных значений. Второй квартиль также называется медианой.
    Третий квартиль (Q3) — это точка на шкале измеренных значений, ниже которой располагаются 75% значений.
    Если данные имеются только в форме порядкового отношения, то качестве меры разброса используется межквартильная широта. Она определяется как

  • Cut points (Точки раздела): Будут вычислены значения процентилей, разделяющие выборку на группы наблюдений, которые имеют одинаковую ширину, то есть включают одно
    и то же количество измеренных значений. По умолчанию предлагается количество групп 10. Если задать, к примеру, 4, то будут показаны квартили, то есть квартили соответствуют процентилям 25, 50 и 75.
    Видно, что число показываемых процентилей на единицу меньше заданного числа групп.

  • Percentile(s) (Процентили): Здесь имеются в виду значения процентилей, определяемые пользователем. Введите значение процентиля в пределах от 0 до 100 и щелкните на кнопке Add (Добавить).
    Повторите эти действия для всех желаемых значений процентилей. Значения в порядке возрастания будут показаны в списке. Например, если ввести значения 25, 50 и 75, то мы получим квартили.
    Можно задавать любые значения процентилей, например, 37 и 83. В первом случае (37) будет показано значение выбранной переменной, ниже которого лежат 37% значений, а во втором случае (83) —
    значение, ниже которого располагаются 83% значений.

В группе Dispersion (Разброс) можно выбрать следующие меры разброса:

  • Std. deviation (Стандартное отклонение) — это мера разброса измеренных величин; оно равно квадратному корню из дисперсии. В интервале шириной, равной удвоенному стандартному отклонению,
    который отложен по обе стороны от среднего значения, располагается примерно 67% всех значений выборки, подчиняющейся нормальному распределению.

  • Variance (Дисперсия) — это квадрат стандартного отклонения и, следовательно, эта характеристика также является мерой разброса измеренных величин. О
    на определяется как сумма квадратов отклонений всех измеренных значений от их среднеарифметического значения, деленная на количество измерений минус 1.

  • Range (Размах) — это разница между наибольшим значением (максимумом) и наименьшим значением (минимумом).

  • Minimum (Минимум) — Наименьшее значение.

  • Maximum (Максимум) — Наибольшее значение.

  • S.E. mean (Стандартная ошибка среднего значения) — В интервале шириной, равной удвоенной стандартной ошибке, отложенному вокруг среднего значения,
    располагается среднее значение генеральной совокупности с вероятностью примерно 67%. Стандартная ошибка определяется как стандартное отклонение, деленное на квадратный корень из объема выборки.

Обычно мерами разброса переменных, относящихся к интервальной шкале и подчиняющихся нормальному распределению, служат стандартное отклонение и стандартная ошибка.
Как было сказано выше, стандартное отклонение позволяет задать диапазон разброса отдельных значений. По так называемому правилу кулака, в одном диапазоне стандартного отклонения
(охватывающем ширину стандартного отклонения в обе стороны от среднего значения) располагается примерно 67% значений, в диапазоне удвоенного стандартного отклонения — примерно 95%,
а в диапазоне утроенного стандартного отклонения — примерно 99% значений.

С другой стороны, стандартная ошибка позволяет задать доверительный интервал для среднего значения. В диапазоне удвоенной стандартной ошибки по обе стороны от среднего значения
с вероятностью примерно 95% находится среднее значение генеральной совокупности. С вероятностью примерно 99% она лежит в диапазоне утроенной стандартной ошибки.
Часто указывают только одну из этих двух мер разброса, обычно — стандартную ошибку, так как ее значение меньше. Во всех случаях следует точно выяснить, какая из мер разброса имеется в виду.

В группе Central Tendency (Средние) можно выбрать следующие характеристики:

  • Mean (Среднее значение) — это арифметическое среднее измеренных значений; оно определяется как сумма значений, деленная на их количество.
    Например, если имеется 12 измеренных значений и их сумма составляет 600, то среднее значение будет х = 600 : 12 = 50.

  • Median (Медиана) — это точка на шкале измеренных значений, выше и ниже которой лежит по половине всех измеренных значений. Например, если измеренные значения таковы:

37854639284,

то сначала они располагаются в порядке возрастания: 23344567889.

В данном случае медианой будет значение 5. Всего у нас 11 измеренных значений, следовательно, медианой является шестое значение. Выше него располагается 5 значений, и ниже — тоже 5.
При нечетном количестве значений медиана всегда будет совпадать с одним из измеренных значений. При четном количестве медиана будет средним арифметическим двух соседних значений.
Например, если имеются следующие измеренные значения:

3445678899

то медиана в этом случае будет равна: (6 + 7) : 2 = 6,5.

  • Mode (Мода) — это значение, которое наиболее часто встречается в выборке. Если одна и та же наибольшая частота встречается у нескольких значений, то выбирается наименьшее из них.

  • Sum (Сумма) — сумма всех значений.

В группе Distribution (Распределение) можно выбрать следующие меры несимметричности распределения:

  • Skewness (Коэффициент асимметрии) — это мера отклонения распределения частоты от симметричного распределения, то есть такого, у которого на одинаковом удалении от среднего значения
    по обе стороны выборки данных располагается одинаковое количество значений. Если наблюдения подчиняются нормальному распределению, то асимметрия равна нулю.
    Для проверки на нормальное распределение можно применять следующее правило: Если асимметрия значительно отличается от нуля, то гипотезу о том, что данные взяты из нормально распределенной
    генеральной совокупности, следует отвергнуть. Если вершина асимметричного распределения сдвинута к меньшим значениям, то говорят о положительной асимметрии, в противоположном случае — об отрицательной.

  • Kurtosis (Коэффициент вариации или эксцесс) — указывает, является ли распределение пологим (при большом значении коэффициента) или крутым. Коэффициент вариации равен нулю,
    если наблюдения подчиняются нормальному распределению. Поэтому для проверки на нормальное распределение можно применять еще одно правило: Если коэффициент вариации значительно отличается от нуля,
    то гипотезу о том, что данные взяты из нормально распределенной генеральной совокупности, следует отвергнуть.

Как правило, для переменных, относящихся к интервальной шкале и подчиняющихся нормальному распределению, в качестве основной характеристики используют среднее значение,
а в качестве меры разброса — стандартное отклонение или стандартную ошибку. Для порядковых или интервальных переменных, не подчиняющихся нормальному распределению —
соответственно медиану или первый и третий квартили. Для переменных относящихся к номинальной шкале, нельзя дать других значимых характеристик кроме моды.

В диалоге есть еще один флажок:

  • Values are group midpoints (Значения являются средними точками групп): Если установить этот флажок, то при вычислении медианы и остальных значений процентилей оценки
    этих характеристик будут определяться для концентрированных данных. Этому вопросу посвящен отдельный раздел.

Для переменной alter (возраст) мы определим следующие характеристики: среднее значение, медиану, моду, квартили, стандартное отклонение, дисперсию, размах, минимум, максимум,
стандартную ошибку, асимметрию и эксцесс. Поступите следующим образом:

  • Выберите в меню команды Analyze (Анализ) / Descriptive Statistics (Дескриптивные статистики) / Frequencies… (Частоты)

  • В диалоге Frequencies щелкните на кнопке Reset (Сброс), чтобы отменить прежние настройки.

  • Перенесите переменную alter в список выходных переменных.

  • Щелкните на кнопке Statistics… (Статистика).

  • В диалоге Frequencies: Statistics установите флажки желаемых характеристик. Затем щелкните на кнопке Continue (Продолжить). Вы вернетесь в диалог Frequencies.

  • В диалоге Frequencies деактивируйте опцию Display frequency tables (Показывать частотные таблицы). Щелкните на кнопке ОК.

В окне просмотра появятся следующие результаты:

Statistics (Статистика)

Alter

NValid (Допустимые)106
Missing (Утерянные)2
Mean (Среднее значение)22,24
Std. Error of Mean (Стандартная ошибка среднего)21
Median (Медиана)22,00
Mode (Мода)21
Std. Deviation (Стандартное отклонение)2,189
Variance (Дисперсия)4,791
Skewness (Асимметрия)0,859
Std. Error of Skewness (Стандартная ошибка асимметрии)0,235
Kurtosis (Коэффициент вариации / Эксцесс)1,042
Std. Error of Kurtosis (Стандартная ошибка эксцесса)0,465
Range (Размах)11
Minimum (Минимум)18
Maximum (Максимум)29
Percentiles (Процентили)2521,00
5022,00
7523,00

Респонденты опроса о психическом состоянии и социальном положении имеют средний возраст 22,24 года. Медиана составляет 22. Большинству респондентов 21 год (это мода).
Самому молодому респонденту 18 лет (минимум), самому старшему — 29 лет (максимум). Самый старший респондент на 11 лет старше самого молодого (размах).
Стандартное отклонение составляет 2,19. Следовательно, дисперсия — квадрат стандартного отклонения — равна (2,19)2 = 4,79.
Асимметрия и коэффициент вариации даны со соответсвующими стандартными ошибками.

CFA — Дисперсия и стандартное отклонение | программа CFA

Среднее абсолютное отклонение позволяет решить проблему, заключающуюся в том, что сумма отклонений от среднего равна нулю. Для этого при расчете среднего используется абсолютное значение отклонений.

Второй подход к расчету отклонений состоит в их возведении в квадрат.

Дисперсия и стандартное отклонение, основанные на квадрате отклонений, являются двумя наиболее широко используемыми мерами дисперсии:

  • Дисперсия определяется как среднее квадратов отклонений от среднего значения.2 \over N } }\)    (формула 11),

    где μ [мю] — это среднее генеральной совокупности, а N — размер генеральной совокупности.

    Зная среднее значение μ, мы можем использовать Формулу 11 для вычисления суммы квадратов отклонений от среднего с учетом всех N элементов в генеральной совокупности, а затем для определения среднего квадратов отклонений путем деления этой суммы на N.

    Независимо от того, является ли отклонение от среднего положительным или отрицательным, возведение в квадрат этой разности дает положительное число.

    Таким образом, дисперсия решает проблему отрицательных отклонений от среднего значения, устраняя их посредством операции возведения в квадрат этих отклонений.


    Рассмотрим пример.

    Прибыль в процентах от выручки для оптовых клубов BJ’s Wholesale Club, Costco и Walmart за 2012 год составляла 0.9%, 1.6% и 3.5% соответственно. Мы рассчитали среднюю прибыль в процентах от выручки как 2.0%.

    Следовательно, дисперсия прибыли в процентах от выручки составляет:

    (1/3)[(0.9 — 2.0)2 + (1.6 — 2.0)2 + (3.5 — 2.0)2]

    = (1/3)(-1.12 + -0.42 + 1.52)

    = (1/3)(1.21 + 0.16 + 2.25) = (1/3)(3.62) = 1.21.

    Стандартное отклонение генеральной совокупности.

    Поскольку дисперсия измеряется в квадратах, нам нужен способ вернуться к исходным единицам. Мы можем решить эту проблему, используя стандартное отклонение, т.е. квадратный корень из дисперсии.

    Стандартное отклонение легче интерпретировать, чем дисперсию, поскольку стандартное отклонение выражается в той же единице измерения, что и наблюдения.

    Формула стандартного отклонения генеральной совокупности.

    Стандартное отклонение генеральной совокупности (или просто стандартное отклонение, а также среднеквадратическое отклонение, от англ.2 \over N} }\)  (формула 12),

    где μ [мю] — это среднее генеральной совокупности, а N — размер генеральной совокупности.


    Используя пример прибыли в процентах от выручки для оптовых клубов BJ’s Wholesale Club, Costco и Walmart за 2012 год, в соответствии с Формулой 12, мы вычислим дисперсию 1.21, а затем возьмем квадратный корень: \( \sqrt{1.21} \) = 1.10.

    Как дисперсия, так и стандартное отклонение являются примерами параметров распределения. В последующих чтениях мы введем понятие дисперсии и стандартного отклонения как меры риска.

    Занимаясь инвестициями, мы часто не знаем среднего значения интересующей совокупности, обычно потому, что мы не можем практически идентифицировать или провести измерения для каждого элемента генеральной совокупности.

    Поэтому мы рассчитываем среднее значение по генеральной совокупности и среднее выборки, взятой из совокупности, и вычисляем выборочную дисперсию или стандартное отклонение выборки, используя формулы, немного отличающиеся от Формул 11 и 12.

    Мы обсудим эти вычисления далее.

    Однако в инвестициях у нас иногда есть определенная группа, которую мы можем считать генеральной совокупностью. Для четко определенных групп наблюдений мы используем Формулы 11 и 12, как в следующем примере.

    Пример расчета стандартного отклонения для генеральной совокупности.

    В Таблице 20 представлен годовой оборот портфеля из 12 фондов акций США, которые вошли в список Forbes Magazine Honor Roll 2013 года.

    Журнал Forbes ежегодно выбирает американские взаимные фонды, отвечающие определенным критериям для своего почетного списка Honor Roll.

    Критериями являются:

    • сохранение капитала (эффективность на медвежьем рынке),
    • непрерывность управления (у фонда должен управлять менеджер непрерывно, в течение не менее 6 лет), диверсификация портфелей,
    • доступность (дисквалификация фондов, которые закрыты для новых инвесторов), и
    • долгосрочные показатели эффективности после уплаты налогов.

    Оборачиваемость или оборот портфеля, показатель торговой активности, является меньшим значением из стоимости продаж или покупок за год, деленным на среднюю чистую стоимость активов за год. Количество и состав списка Forbes Honor Roll меняются из года в год.














    Таблица 20. Оборот портфеля: взаимные фонды Forbes Honor Roll за 2013 год.

    Фонд

    Годовой оборот портфеля (%)

    Bruce Fund (BRUFX)

    10

    CGM Focus Fund (CGMFX)

    360

    Hotchkis And Wiley Small Cap Value A Fund (HWSAX)

    37

    Aegis Value Fund (AVALX)

    20

    Delafield Fund (DEFIX)

    49

    Homestead Small Company Stock Fund (HSCSX)

    1

    Robeco Boston Partners Small Cap Value II Fund (BPSCX)

    32

    Hotchkis And Wiley Mid Cap Value A Fund (HWMAX)

    72

    T Rowe Price Small Cap Value Fund (PRSVX)

    9

    Guggenheim Mid Cap Value Fund Class A (SEVAX)

    19

    Wells Fargo Advantage Small Cap Value Fund (SSMVX)

    16

    Stratton Small-Cap Value Fund (STSCX)

    11

    Источник: Forbes (2013).2 \over N } \), сначала рассчитав числитель, а затем разделив результат на N = 12.

    Числитель (сумма квадратов отклонений от среднего) равен:

    (10 — 53)2 + (360 — 53)2 + (37 — 53)2 + (20 — 53)2 +

    (49 — 53)2 + (1 — 53)2 + (32 — 53)2 + (72 — 53)2 +

    (9 — 53)2 + (19 — 53)2 + (16 — 53)2 + (11 — 53)2 = 107,190

    Таким образом, σ2 = 107,190/12 = 8,932.50.

    Для расчета стандартного отклонения находим квадратный корень:

    \( \sigma = \sqrt{ 8,932.50 } \) = 94.51%.

    Единицей измерения дисперсии является процент в квадрате, поэтому единицей измерения стандартного отклонения также является процент.


    Решение для части 3:

    Если генеральная совокупность четко определена как фонды Forbes Honor Roll за один конкретный год (2013 г.), и если под оборотом портфеля понимается конкретный одногодичный период, о котором отчитывается Forbes, то применение формул генеральной совокупности для дисперсии и стандартного отклонения уместно.

    Результаты 8,932.50 и 94.51 представляют собой, соответственно, перекрестную дисперсию и стандартное отклонение годового оборота портфеля для фондов Forbes Honor Roll за 2013 год.

    Фактически, мы не могли должным образом использовать фонды Honor Roll для оценки дисперсии оборота портфеля (например) любой другой по-разному определенной генеральной совокупности, потому что фонды Honor Roll не являются случайной выборкой из какой-либо большей генеральной совокупности взаимных фондов США.

    Выборочная дисперсия.

    Во многих случаях в управлении инвестициями подгруппа или выборка из генеральной совокупности — это все, что мы можем наблюдать. Когда мы имеем дело с выборками, сводные показатели называются статистикой.

    Статистика, которая измеряет дисперсию по выборке, называется выборочной дисперсией или дисперсией выборки (англ. ‘sample variance’).

    В приведенном ниже обсуждении обратите внимание на использование латинских букв вместо греческих для обозначения объема выборки.2 \).

  • Разделить сумму квадратов отклонений от среднего на (n — 1).
  • Мы проиллюстрируем расчет выборочной дисперсии и выборочного стандартного отклонения на примере ниже.

    Отличие выборочной дисперсии от дисперсии генеральной совокупности.

    Мы используем обозначение s2 для выборочной дисперсии, чтобы отличить ее от дисперсии генеральной совокупности σ2.

    Формула для выборочной дисперсии почти такая же, как и для дисперсии генеральной совокупности, за исключением использования среднего значения выборки \( \overline X \) вместо среднего значения генеральной совокупности μ и другого делителя.

    В случае дисперсии генеральной совокупности мы делим числитель на размер совокупности N. Однако для дисперсии выборки мы делим ее на размер выборки минус 1 или n — 1. Используя n — 1 (а не n) в качестве делителя мы улучшаем статистические свойства выборочной дисперсии.

    В статистических терминах выборочная дисперсия, определенная в Формуле 13, является несмещенной оценкой (англ. ‘unbiased estimator ‘) дисперсии генеральной совокупности σ2.

    Мы обсудим эту концепцию далее в чтении о выборке.

    Величина n — 1 также называется числом степеней свободы (англ. ‘number of degrees of freedom’) при оценке дисперсии генеральной совокупности. Чтобы оценить дисперсию s2, мы должны сначала вычислить среднее. После того как мы вычислили среднее значение выборки, существует только n — 1 независимых отклонений от него.

    Стандартное отклонение выборки.

    Для стандартного отклонения генеральной совокупности мы аналогичным образом можем вычислить стандартное отклонение выборки, взяв квадратный корень из положительной дисперсии выборки.

    Формула стандартного отклонения выборки.

    Стандартное отклонение выборки (выборочное стандартное отклонение, выборочное среднеквадратическое отклонение, англ. ‘sample standard deviation’), обозначается символом s и рассчитывается следующим образом:

    \(\mathbf{ s = \sqrt{ \sum_{i=1}^{n} ( X_i — \overline X )^2 \over n-1 } }\)  (формула 14),


    где \( \overline X \) — среднее значение выборки, а n — количество наблюдений в выборке.

    Чтобы рассчитать стандартное отклонение выборки, мы сначала вычисляем дисперсию выборки, используя приведенные выше шаги. Затем мы берем квадратный корень из выборочной дисперсии.

    Пример, приведенный ниже, иллюстрирует расчет выборочной дисперсии и стандартного отклонения выборки для двух взаимных фондов, представленных ранее.

    Пример расчета выборочной дисперсии и стандартного отклонения выборки.

    После расчета геометрических и арифметических средних доходностей двух взаимных фондов в Примере (1) мы вычислили две меры дисперсии для этих фондов, размах и среднее абсолютное отклонение доходности (см.  Пример расчета размаха и среднего абсолютного отклонения для оценки риска).

    Теперь мы вычислим выборочную дисперсию и стандартное отклонение выборки для доходности тех же двух фондов.







    Таблица 15. Совокупная доходность двух взаимных фондов, 2008-2012 гг.

    (повтор).

    Год

    Фонд Selected

    American Shares

    (SLASX)

    Фонд T. Rowe Price

    Equity Income

    (PRFDX)

    2008

    -39.44%

    -35.75%

    2009

    31.64

    25.62

    2010

    12.53

    15.15

    2011

    -4.35

    -0.72

    2012

    12.82

    17.25

    Источник: performance.morningstar.com.

    На основании приведенных выше данных сделайте следующее:

    1. Рассчитайте выборочную дисперсию доходности для (A) SLASX и (B) PRFDX.
    2. Рассчитайте выборочное стандартное отклонение доходности для (A) SLASX и (B) PRFDX.
    3. Сравните дисперсию доходности, измеренную стандартным отклонением доходности и средним абсолютным отклонением доходности для каждого из двух фондов.

    Решение для части 1:

    Чтобы вычислить выборочную дисперсию, мы используем Формулу 13 (значения отклонений приведены в процентах).

    А. SLASX:

    1. Среднее значение выборки:

    \( \overline R \) = (-39.44 + 31.64 + 12.53 — 4.35 +12.82)/ 5 =

    13.20/5 = 2.64%.

    2. Квадратичные отклонения от среднего значения:

    (-39.44 — 2.64)2 = (-42.08)2 = 1,770.73

    (31.64 — 2.64)2 = (29.00)2 = 841.00

    (12.53 — 2.64)2 = (9.89)2 = 97.81

    (-4.35 — 2.64)2 = (-6.99)2 = 48.86

    (12.82 — 2.64)2 = (10.18)2 = 103.63

    3. Сумма квадратов отклонений от среднего составляет:

    1,770.73 + 841.00 + 97.81 + 48.86 + 103.63 = 2,862.03.

    4. Разделим сумму квадратов отклонений от среднего на (n — 1):

    2,862.03/(5 — 1) = 2,862.03/4 = 715.51

    B. PRFDX:

    1. Среднее значение выборки:

    \( \overline R \) = (-35.75 + 25.62 + 15.15 — 0.72 + 17.25)/5 = 21.55/5 = 4.31%.

    2. Квадратичные отклонения от среднего значения:

    (-35.75 — 4.31)2 = (-40.06)2 = 1,604.80

    (25.62 — 4.31)2 = (21.31)2 = 454.12

    (15.15 — 4.31)2 = (10.84)2 = 117.51

    (-0.72 — 4.31)2 = (-5.03)2 = 25.30

    (17.25 — 4.31)2 = (12.94)2 = 167.44

    3. Сумма квадратов отклонений от среднего составляет:

    1,604.80 + 454.12 + 117.51 + 25.30 + 167.44 = 2,369.17.

    4. Разделим сумму квадратов отклонений от среднего на (n — 1):

    2,369.17/4 = 592.29


    Решение для части 2:

    Чтобы найти стандартное отклонение, мы берем положительный квадратный корень из дисперсии.

    A. Для SLASX, s = \( \sqrt 715.51 \) = 26.7%.

    B. Для PRFDX, s = \( \sqrt 592.29 \) = 24.3%.


    Решение для части 3:

    Таблица 21 суммирует результаты части 2 для стандартного отклонения и включает результаты для MAD из Примера расчета размаха и среднего абсолютного отклонения для оценки риска.




    Таблица 21. Два взаимных фонда: сравнение стандартного отклонения и среднего абсолютного отклонения (MAD).

    Фонд

    Стандартное

    отклонение (%)

    Среднее

    абсолютное

    отклонение (%)

    SLASX

    26.7

    19.6

    PRFDX

    24.3

    18.0

    Обратите внимание, что среднее абсолютное отклонение меньше стандартного отклонения. Среднее абсолютное отклонение всегда будет меньше или равно стандартному отклонению, потому что стандартное отклонение придает больший вес большим отклонениям, чем маленьким (помните, что отклонения возводятся в квадрат).

    Поскольку стандартное отклонение является мерой дисперсии относительно среднего арифметического, мы обычно представляем  среднее арифметическое и стандартное отклонение вместе при анализе данных.

    Когда мы имеем дело с данными, которые представляют собой временной ряд процентных изменений, представление геометрического среднего, представляющего собой сложную ставку скорости роста, также очень полезно.


    В таблице 22 представлены исторические геометрические и арифметические средние доходности, а также историческое стандартное отклонение доходности для годовой и месячной доходности S&P 500.

    Мы представляем эту статистику для номинальной (без поправки на инфляцию) доходности, чтобы мы могли наблюдать первоначальные величины доходности.




    Таблица 22. Доходность рынка ценных бумаг: средние значения и стандартные отклонения.

    Ставка доходности

    Геометрическое

    среднее (%)

    Среднее

    арифметическое (%)

    Стандартное отклонение

    S&P 500 (Годовая)

    9.84

    11.82

    20.18

    S&P 500 (Месячная)

    0.79

    0.94

    5.50

    Источник: Ibbotson.

    Формулы стандартного отклонения

    Отклонение просто означает, насколько далеко от нормы

    Стандартное отклонение

    Стандартное отклонение — это показатель того, насколько разброс
    наши номера
    .

    Вы можете сначала прочитать эту более простую страницу о стандартном отклонении.

    Но здесь мы объясняем формулы .

    Символ стандартного отклонения — σ (греческая буква сигма).

    Это формула для стандартного отклонения:

    Сказать что? Объясните, пожалуйста!

    ОК. Давайте объясним это шаг за шагом.

    Допустим, у нас есть набор чисел вроде 9, 2, 5, 4, 12, 7, 8, 11.

    Чтобы вычислить стандартное отклонение этих чисел:

    • 1. Вычислите среднее (простое среднее
      номеров)
    • 2. Затем для каждого числа: вычтите Среднее и возведите результат в квадрат
    • 3.Затем вычислите среднее значение из этих квадратов разностей.
    • 4. Извлеките из этого квадратный корень, и все готово!

    Формула действительно говорит обо всем этом, и я покажу вам, как это сделать.

    Объяснение формулы

    Во-первых, у нас есть несколько примеров значений для работы:

    Пример: У Сэма 20 кустов роз.

    Количество цветков на каждом кусте

    9, 2, 5, 4, 12, 7, 8, 11, 9, 3, 7, 4, 12, 5, 4, 10, 9, 6, 9, 4

    Определите стандартное отклонение.

    Шаг 1. Определите среднее значение

    В приведенной выше формуле μ (греческая буква «mu») — это среднее всех наших значений …

    Пример: 9, 2, 5, 4, 12, 7, 8, 11, 9, 3, 7, 4, 12, 5, 4, 10, 9, 6, 9, 4

    Среднее значение:

    9 + 2 + 5 + 4 + 12 + 7 + 8 + 11 + 9 + 3 + 7 + 4 + 12 + 5 + 4 + 10 + 9 + 6 + 9 + 4
    20

    =

    140
    20
    = 7

    Итак:

    мк = 7

    Шаг 2.Затем для каждого числа: вычтите Среднее и возведите результат в квадрат

    Это часть формулы, которая гласит:

    Итак, что такое x и ? Это отдельные значения x 9, 2, 5, 4, 12, 7 и т. Д.

    Другими словами x 1 = 9, x 2 = 2, x 3 = 5 и т. Д.

    Итак, он говорит: «для каждого значения вычтите среднее и возведите результат в квадрат», например,

    Пример (продолжение):

    (9-7) 2 = (2) 2 = 4

    (2-7) 2 = (-5) 2 = 25

    (5-7) 2 = (-2) 2 = 4

    (4-7) 2 = (-3) 2 = 9

    (12-7) 2 = (5) 2 = 25

    (7-7) 2 = (0) 2 = 0

    (8-7) 2 = (1) 2 = 1

    … и т.д …

    И получаем такие результаты:

    4, 25, 4, 9, 25, 0, 1, 16, 4, 16, 0, 9, 25, 4, 9, 9, 4, 1, 4, 9

    Шаг 3. Затем вычислите среднее значение квадратов разностей.

    Чтобы вычислить среднее значение, сложите все значения , затем разделите на сколько .

    Сначала сложите все значения из предыдущего шага.

    Но как сказать в математике «сложить все»? Используем «Сигма»: Σ

    Удобная сигма-нотация позволяет суммировать столько терминов, сколько мы хотим:

    Сигма-нотация

    Мы хотим сложить все значения от 1 до N, где в нашем случае N = 20, потому что имеется 20 значений:

    Пример (продолжение):

    Это означает: суммировать все значения от (x 1 -7) 2 до (x N -7) 2

    Мы уже вычислили (x 1 -7) 2 = 4 и т. Д.на предыдущем шаге, так что просто суммируйте их:

    = 4 + 25 + 4 + 9 + 25 + 0 + 1 + 16 + 4 + 16 + 0 + 9 + 25 + 4 + 9 + 9 + 4 + 1 + 4 + 9 = 178

    Но это еще не среднее значение, нам нужно разделить на сколько , что получается путем умножения на 1 / N (то же самое, что деление на N):

    Пример (продолжение):

    Среднее значение квадратов разностей = (1/20) × 178 = 8,9

    (Примечание: это значение называется «Дисперсия»)

    Шаг 4.Извлеките квадратный корень из этого:

    Пример (заключение):

    σ = √ (8,9) = 2,983 …

    СДЕЛАНО!

    Стандартное отклонение выборки

    Но подождите, это еще не все …

    … иногда наши данные — это всего лишь выборка всего населения.

    Пример: У Сэма

    20 кустов роз , но посчитал цветов только на 6 из них !

    «Население» — всего 20 кустов роз,

    , а «образец» — это 6 кустов, цветы которых Сэм считал.

    Допустим, у Сэма количество цветов:

    9, 2, 5, 4, 12, 7

    Мы все еще можем оценить стандартное отклонение.

    Но когда мы используем выборку в качестве оценки для всей совокупности , формула стандартного отклонения изменяется на:

    Формула для Стандартное отклонение выборки :

    Важным изменением является «N-1» вместо «N» (что называется «поправкой Бесселя»).

    Символы также меняются, чтобы отразить, что мы работаем над выборкой, а не со всей генеральной совокупностью:

    • Среднее значение теперь x (для выборочного среднего) вместо μ (среднее значение генеральной совокупности),
    • И ответ: с (для стандартного отклонения выборки) вместо σ .

    Но на расчеты это не влияет. Только N-1 вместо N меняет вычисления.

    Хорошо, давайте теперь вычислим стандартное отклонение выборки :

    Шаг 1.Найдите среднее значение

    Пример 2: Использование значений выборки 9, 2, 5, 4, 12, 7

    Среднее значение: (9 + 2 + 5 + 4 + 12 + 7) / 6 = 39/6 = 6,5

    Итак:

    х = 6,5

    Шаг 2. Затем для каждого числа: вычтите Среднее и возведите результат в квадрат.

    Пример 2 (продолжение):

    (9 — 6,5) 2 = (2,5) 2 = 6,25

    (2 — 6,5) 2 = (-4,5) 2 = 20,25

    (5 — 6,5) 2 = (-1.5) 2 = 2,25

    (4 — 6,5) 2 = (-2,5) 2 = 6,25

    (12 — 6,5) 2 = (5,5) 2 = 30,25

    (7 — 6,5) 2 = (0,5) 2 = 0,25

    Шаг 3. Затем вычислите среднее значение квадратов разностей.

    Чтобы вычислить среднее значение, сложите все значения , затем разделите на сколько .

    Но подождите … мы вычисляем стандартное отклонение Sample , поэтому вместо деления на количество (N) мы разделим на N-1

    Пример 2 (продолжение):

    Сумма = 6.25 + 20,25 + 2,25 + 6,25 + 30,25 + 0,25 = 65,5

    Разделить на N-1 : (1/5) × 65,5 = 13,1

    (Это значение называется «Выборочная дисперсия»)

    Шаг 4. Извлеките квадратный корень из этого:

    Пример 2 (завершение):

    с = √ (13,1) = 3,619 …

    СДЕЛАНО!

    Сравнение

    Когда мы использовали всю популяцию , мы получили: Среднее = 7 , Стандартное отклонение = 2.983 …

    Когда мы использовали образец , мы получили: Среднее значение выборки = 6,5 , стандартное отклонение выборки = 3,619 …

    Наше среднее значение по выборке было неверным на 7%, а стандартное отклонение по выборке было неверным на 21%.

    Зачем нужно брать образец?

    В основном потому, что так проще и дешевле.

    Представьте, что вы хотите знать, что думает вся страна … вы не можете спрашивать миллионы людей, поэтому вместо этого вы спрашиваете, может быть, 1000 человек.

    Есть хорошая цитата (возможно, Самуэль Джонсон):

    «Не обязательно есть животное целиком, чтобы знать, что мясо жесткое.»

    Это основная идея отбора проб. Чтобы получить информацию о совокупности (такую ​​как среднее значение и стандартное отклонение), нам не нужно смотреть на всех членов совокупности; нам нужен только образец.

    Но когда мы берем образец, мы теряем некоторую точность.

    Сводка

    Население Стандартное отклонение:

    Образец Стандартное отклонение:

    Простое определение, пошаговое видео

    Содержание: Стандартное отклонение (щелкните, чтобы перейти в раздел):

    Основы:

    1. Стандартное отклонение Определение
    2. Как Найти стандартное отклонение вручную

    Дополнительные темы:

    1. Стандартное отклонение для бинома
    2. Стандартное отклонение дискретной случайной величины
    3. Стандартное отклонение для частотного распределения

    Использование технологии:

    1. Найдите стандартное отклонение в Minitab
    2. Найдите стандартное отклонение в SPSS
    3. в Excel
    4. TI-89 Инструкции

    Статьи по теме:

    1. Абсолютное стандартное отклонение

    Стандартное отклонение — это мера разброса в статистике.«Разброс» показывает, насколько разбросаны ваши данные. В частности, он показывает, насколько ваши данные разбросаны относительно среднего или среднего значения. Например, все ли ваши оценки близки к среднему? Или много баллов намного выше (или намного ниже) среднего балла?

    Как это выглядит на графике?

    Кривая колокола (то, что статистики называют «нормальным распределением») обычно рассматривается в статистике как инструмент для понимания стандартного отклонения.

    Следующий график нормального распределения представляет большой объем данных в реальной жизни.Среднее или среднее значение представлено греческой буквой μ в центре. Каждый сегмент (от темно-синего до светло-синего) представляет собой одно стандартное отклонение от среднего. Например, 2σ означает два стандартных отклонения от среднего.

    Пример из реальной жизни

    Кривая нормального распределения может отображать сотни ситуаций в реальной жизни. Вы когда-нибудь замечали в классе, что большинство учеников получают четверку, а некоторые — пятерку или пятерку? Это можно смоделировать с помощью колоколообразной кривой.Вес, рост, привычки питания и режим упражнений людей также можно смоделировать с помощью графиков, подобных этому. Эти знания позволяют компаниям, школам и правительствам делать прогнозы относительно будущего поведения. Для поведения, которое соответствует этому типу колоколообразной кривой (например, успеваемость на SAT), вы сможете предсказать, что 34,1 + 34,1 = 68,2% учащихся наберут очень близко к среднему баллу, или на одно стандартное отклонение от нормы. иметь в виду.

    Когда вы проводите эксперимент (или тест, или опрос), вы обычно работаете с выборкой — небольшой частью генеральной совокупности.Формула для определения стандартного отклонения при работе с образцами:

    Знак Σ в формуле означает «сложить» (см .: Сигма-нотация). Чтобы решить формулу,

    1. Сложите числа,
    2. пл. Им.
    3. Затем разделите.

    Звучит просто, но становится утомительно при работе с большими размерами выборки (потому что вам придется складывать и возводить в квадрат несколько раз). В приведенном ниже примере задачи всего 9 точек данных, но она должна дать вам хороший пример того, насколько утомительными могут быть ручные вычисления.Если вам все же приходится рассчитывать его вручную (для домашнего задания или теста), обязательно используйте калькулятор, чтобы проверить свой ответ.

    Пример проблемы:

    Q. Найдите стандартное отклонение для следующих результатов:
    {12, 15, 17, 20, 30, 31, 43, 44, 54}

    Шаг 1: сложите числа:
    12 + 15 + 17 + 20 + 30 + 31 + 43 + 44 + 54 = 266.

    Шаг 2: возведите ответ из шага 1 в квадрат:
    266 x 266 = 70756

    Шаг 3: Разделите свой ответ из шага 2 на количество предметов ( n ) в вашем наборе.В этом примере у нас 9 элементов, поэтому:
    70756/9 = 7861.777777777777 (деление на n)

    Отложите это число на мгновение. Он понадобится вам позже.

    Шаг 4. Возведите исходные числа {12, 15, 17, 20, 30, 31, 43, 44, 54} в квадрат по одному, затем сложите их:

    (12 x 12) + (15 x 15) + (17 x 17) + (20 x 20) + (30 x 30) + (31 x 31) + (43 x 43) + (44 x 44) + ( 54 x 54) = 9620

    Шаг 5: вычтите шаг 4 из шага 3.

    9620 — 7861.777777777777 = 1758.2222222222226

    Обратите внимание, что я еще не округляю. Вы должны сохранить все десятичные разряды до самого конца, затем можете округлить. Округление в середине приведет к тому, что ваш ответ будет неправильным, чтобы получить неправильный ответ из учебника. Отложите это число на мгновение.

    Шаг 6: Вычтите 1 из n . У нас 9 элементов, поэтому n = 9:

    .

    9 — 1 = 8

    Шаг 7: Разделите шаг 5 на шаг 6, чтобы получить отклонение :
    1758,2222222222226 / 8 = 219.77777777777783

    Шаг 8: Извлеките квадратный корень из шага 7:
    √ (219,77777777777783) = 14,824

    1958058
    Стандартное отклонение составляет 14,825.

    В начало

    нужна помощь с домашним заданием? Посетите нашу страницу обучения!

    (Щелкните, чтобы перейти к разделу)
    Стандартное отклонение для бинома: TI-83
    Стандартное отклонение для бинома: вручную

    TI 83 Стандартное отклонение для бинома

    В TI 83 нет встроенной функции для определения стандартного отклонения бинома.Вы должны ввести уравнение вручную.

    Пример задачи : Найдите стандартное отклонение для биномиального распределения с n = 5 и p = 0,12.

    Шаг 1: Вычтите p из 1, чтобы найти q.
    1 — 0,12 ВВОД
    = 0,88

    Шаг 2: Умножить n раз p на q.
    5 * .12 * .88 ВВЕДИТЕ
    = .528

    Шаг 3: Найдите квадратный корень из ответа на шаге 2.
    √.528 = =.727 (округлено до 3 знаков после запятой).

    Стандартное отклонение для бинома: вручную

    Подбрасывание монеты может быть биномиальным экспериментом.

    Биномиальное распределение — это один из простейших типов распределений в статистике. Это тип распределения, в котором есть либо успех, либо неудача. Например, выиграть в лотерею: или не выиграть в лотерею. Вы можете найти стандартное отклонение для биномиального распределения двумя способами:

    1. С формулой
    2. С таблицей распределения вероятностей (шаги прокрутите вниз)

    Формула для определения стандартного отклонения для биномиального распределения:

    Посмотрите видео или прочтите следующие шаги:

    Пример вопроса:

    Найдите стандартное отклонение для следующего биномиального распределения: подбросьте монетку 1000 раз, чтобы узнать, сколько орлов вы получите.

    Шаг 1: Определите n и p из вопроса. N — количество попыток (дано как 1000), а p — вероятность, равная 0,5 (у вас есть 50% шанс получить орел при любом подбрасывании монеты).

    На этом этапе вы можете вставить эти числа в формулу и решить. Если формулы не ваша сильная сторона, выполните следующие дополнительные действия:

    Шаг 2: Умножьте n на p:
    1000 * 0,5 = 500.

    Шаг 3: Вычтите «p» из 1:
    1 — 0,5 = 0,5.

    Шаг 4: Умножьте Шаг 2 на Шаг 3: 500 *.5 = 250,

    Шаг 5: Извлеките квадратный корень из шага 4:
    √ 250 = 15,81.

    Вот и все!

    При использовании дискретных случайных величин иногда вместо «p» и «n» предоставляется таблица распределения вероятностей. Если у вас есть таблица, вы можете рассчитать стандартное отклонение дискретных случайных величин по этой формуле:

    Пример вопроса: Найдите стандартное отклонение дискретных случайных величин, показанных в следующей таблице, которые представляют собой подбрасывание трех монет:

    Шаг 1: Найдите среднее значение (это также называется ожидаемым значением), умножив вероятности на x в каждом столбце и сложив их все:
    μ = (0 * 0.125) + (1 * 0,375) + (2 * 0,375) + (3 * 0,125) = 1,5

    Шаг 2: обработайте внутреннюю часть приведенного выше уравнения без квадратного корня:

    • ((0 — 1,5) 2 * 0,125) +
    • ((1 — 1,5) 2 * 0,375) +
    • ((2 — 1,5) 2 * 0,375) +
    • ((3 — 1,5) 2 * 0,125) +
    • = 0,75

    Шаг 3: Извлеките квадратный корень из Шага 2:
    σ = √ 0,75 = 0,8660254.

    Вот и все!

    В начало

    В начало
    Формула для определения стандартного отклонения для частотного распределения:

    Где:

    • μ — среднее значение для частотного распределения,
    • f — индивидуальные отсчеты частоты,
    • x — значение, связанное с частотами.

    Если формулы не ваша сильная сторона, посмотрите это короткое видео, в котором показано, как работать с формулой:

    Посмотрите видео или выполните следующие действия:

    Пример вопроса: Найдите стандартное отклонение в Minitab для следующих данных: 102, 104, 105, 110, 112, 116, 124, 124, 125, 240, 245, 254, 258, 259, 265, 265, 278 , 289, 298, 311, 321, 321, 324, 354

    Шаг 1: Введите данные в один столбец на листе Minitab.

    Шаг 2: Нажмите «Статистика», затем нажмите «Базовая статистика», затем нажмите «Описательная статистика».

    Шаг 3: Выберите переменные, для которых требуется найти стандартное отклонение для , а затем нажмите «Выбрать», чтобы переместить имена переменных в правое окно.

    Шаг 4: Щелкните кнопку «Статистика».

    Шаг 5: Установите флажок «Стандартное отклонение» , а затем дважды нажмите «ОК». Стандартное отклонение будет отображаться в новом окне.

    Вот и все!

    В начало

    Инструмент для вычисления стандартного отклонения в SPSS находится в разделе «Аналитика> Описательная статистика» на панели инструментов. Вы также можете использовать опцию «Частоты» в том же меню. На видео ниже показаны оба варианта, или прочтите ниже шаги только с первым вариантом.

    Если вы уже ввели свои данные в рабочий лист, переходите к шагу 3.

    Шаг 1: Откройте новый рабочий лист для ввода данных. Когда откроется SPSS, выберите переключатель «введите данные» справа от диалогового окна «Что бы вы хотели сделать».

    Шаг 2: Введите данные в рабочий лист. Вы можете использовать любое количество столбцов для ввода данных, но не оставляйте пустых строк между данными.

    Шаг 3: Щелкните «Анализировать» на панели инструментов и затем наведите указатель мыши на «Описательная статистика». Щелкните «Descriptives», чтобы открыть диалоговое окно переменных .

    Шаг 4: Выберите переменные, для которых требуется найти описательную статистику для .SPSS необходимо знать, где находятся данные, для которых вы хотите рассчитать стандартное отклонение. Система заполнит левое поле возможными вариантами (столбцы данных, которые вы ввели), но вам нужно будет выбрать, какие переменные вы хотите включить, и перенести эти списки в правое поле. Чтобы передать списки, щелкните центральную стрелку, чтобы переместить эти переменные из левого поля в правое.

    Шаг 5: Отметьте поле «Стандартное отклонение», затем нажмите «ОК». Ответ отобразится справа от окна, в последнем столбце, озаглавленном «Стандартное отклонение.”

    В начало

    Содержание:
    Excel 2013 и новее
    Excel 2007/2010
    STDEV или STDEV.P?

    Примечания для Mac:
    Для стандартного отклонения для всей совокупности (σ) используйте:
    STDEV.P (A1: A10)
    Для стандартного отклонения выборки (часть генеральной совокупности) используйте:
    STDEV .S (A1: A10)

    Excel 2013

    Посмотрите видео или прочтите ниже:

    Стандартное отклонение можно определить двумя разными способами:

    1. Функция STDEV.
    2. Пакет инструментов анализа данных.

    Рассмотрите возможность установки пакета инструментов анализа данных, особенно если вы собираетесь выполнять несколько анализов своих данных.

    1. Функция STDEV

    Шаг 1: Введите данные в один столбец. Например, столбец A.

    Шаг 2: Щелкните любую пустую ячейку.

    Шаг 3: Введите «= СТАНДОТКЛОН (A1: A99)», где A1: A99 — расположение ячеек ваших данных.

    Шаг 4: Нажмите «ОК».

    2. Пакет инструментов

    Шаг 1: Щелкните вкладку «Данные», затем щелкните «Анализ данных».”

    Шаг 2: Щелкните «Описательная статистика», затем щелкните «ОК».

    Шаг 3: Щелкните поле Диапазон ввода , затем введите, где находятся ваши данные. Например, если вы ввели данные в ячейки от B1 до B50, введите в поле «B1: B50».

    Шаг 4. Выберите переключатель «Строки» или «Столбцы». Это зависит от того, как вы вводите свои данные.

    Шаг 5. Щелкните поле «Ярлыки в первой строке», если у ваших данных есть заголовки столбцов. Заголовок столбца — это первое поле в столбце (например,грамм. A1, A2, A3…), на котором есть какой-то ярлык, например «кошки», «образец» или «луны».

    Шаг 6: Установите флажок «Описательная статистика».

    Шаг 7: Выберите место для ваших результатов. Например, щелчок по переключателю «Новый рабочий лист» выведет ваши результаты на новый рабочий лист.

    Шаг 8: Нажмите «ОК».

    В начало

    Excel 2007

    Шаги по существу те же для Excel 2007/2010/2013. В этом видео показано, как использовать функцию STDDEV.

    STDEV, STDEV.P, STDEV.S, STDEVA, STDEVPA и STDEVP

    Посмотрите видео или прочтите ниже:

    Excel 2013 имеет ШЕСТЬ функций для стандартного отклонения:

    • СТАНДОТКЛОН,
    • СТАНДАРТ.П,
    • СТАНДОТКЛОН.S,
    • СТАНДОТКЛОН,
    • STDEVPA
    • СТАНДОТКЛОНП.

    Какую функцию вы выберете, зависит от того,:

    1. Вы работаете с выборками или популяциями.
    2. Вы хотите оценить числовые данные или другие типы данных (например, двоичные ИСТИНА и ЛОЖЬ).

    В таблице ниже показаны различия между шестью типами.

    ВЫБОРКА / НАСЕЛЕНИЕ ТИП ДАННЫХ СОВМЕСТИМОСТЬ *? ФУНКЦИЯ
    ОБРАЗЕЦ ЧИСЛОВЫЕ НЕТ = СТАНДАРТ.S
    ОБРАЗЕЦ ЧИСЛОВЫЕ ДА = СТАНДОТКЛОН
    ОБРАЗЕЦ ОБА НЕТ = СТАНДОТКЛОН
    НАСЕЛЕНИЕ ЧИСЛОВЫЕ НЕТ = СТАНДОТКЛОН.P
    НАСЕЛЕНИЕ ЧИСЛОВЫЕ ДА = СТАНДОТКЛОН
    НАСЕЛЕНИЕ ОБА НЕТ = СТАНДАРТНЫЙ ПАРАМЕТР

    * В более ранних версиях Excel.

    Вернуться к началу

    Посмотрите видео или прочтите ниже.

    Пример проблемы: Какое стандартное отклонение для этого списка? 1, 34, 56, 89, 287, 598, 1001.

    Шаг 1: Нажмите HOME.

    Шаг 2: Нажмите КАТАЛОГ .
    Он расположен под клавишей APPS в верхней средней части клавиатуры.

    Шаг 3: Прокрутите до stdDev (.
    Нажмите ENTER.

    Шаг 4: Нажмите 2nd, затем (.
    На дисплее должно отображаться:
    stdDev ({
    Обратите внимание на фигурную скобку:

    Шаг 5: Введите числа . Обязательно вводите запятые после каждого числа.
    Конечный результат должен выглядеть так:
    stdDev ({1,34,56,89,287,598,1001

    Шаг 6: Нажмите 2-й, затем) дважды.
    Это закрывает выражение:
    stdDev ({1,34,56,89,287,598,1001}).

    Шаг 7: Нажмите ENTER , чтобы получить решение:
    375,149.

    В начало

    Список литературы

    Гоник, Л. (1993). Мультяшный справочник по статистике. HarperPerennial.
    Кенни, Дж. Ф. и Кепинг, Э. С. Математика статистики, Pt. 2, 2-е изд. Princeton, NJ: Van Nostrand, 1951.
    Kotz, S .; и др., ред. (2006), Энциклопедия статистических наук, Wiley.
    Папулис, А. Вероятность, случайные величины и случайные процессы, 2-е изд. Нью-Йорк: McGraw-Hill, стр. 144-145, 1984.
    Vogt, W.P. (2005). Словарь статистики и методологии: нетехническое руководство для социальных наук. МУДРЕЦ.
    Линдстрем, Д. (2010). Краткое изложение статистики Шаума, второе издание (Schaum’s Easy Outlines), 2-е издание. McGraw-Hill Education

    ————————————————— —————————-

    Нужна помощь с домашним заданием или контрольным вопросом? С помощью Chegg Study вы можете получить пошаговые ответы на свои вопросы от эксперта в данной области.Ваши первые 30 минут с репетитором Chegg бесплатны!

    Комментарии? Нужно опубликовать исправление? Пожалуйста, оставьте комментарий на нашей странице в Facebook .

    2. Среднее и стандартное отклонение

    Медиана известна как мера местоположения; то есть он сообщает нам, где находятся данные. Как указано в, нам не нужно знать все точные значения для вычисления медианы; если бы мы сделали наименьшее значение еще меньшим, а наибольшее — еще большим, это не изменило бы значение медианы.Таким образом, медиана не использует всю информацию, содержащуюся в данных, и поэтому можно показать, что она менее эффективна, чем среднее или среднее значение, которое действительно использует все значения данных. Чтобы вычислить среднее значение, мы складываем наблюдаемые значения и делим их на их количество. Сумма значений, полученных в таблице 1.1, составила 22,5, которые были разделены на их количество, 15, и получили среднее значение 1,5. Этот знакомый процесс —
    .
    удобно выражается следующими символами: (произносится как «x bar») означает среднее значение; x — каждое из значений свинца в моче; n — количество этих значений; а σ, греческая заглавная сигма (наша буква «S») означает «сумму».Главный недостаток среднего — то, что он чувствителен к удаленным точкам. Например, замена 2,2 на 22 в таблице 1.1 увеличивает среднее значение до 2,82, в то время как медиана останется неизменной. Помимо показателей местоположения, нам нужны меры того, насколько изменчивы данные. Мы встретили два из этих критериев, диапазон и межквартильный размах, в главе 1.

    Диапазон — важное измерение, поскольку цифры вверху и внизу обозначают результаты, наиболее далекие от общности. Однако они не дают большого представления о разбросе наблюдений за средним значением.Именно здесь на помощь приходит стандартное отклонение (SD).

    Теоретическая основа стандартного отклонения сложна и не должна беспокоить обычного пользователя. Мы обсудим выборку и совокупности в главе 3. Практический момент, который следует отметить здесь, заключается в том, что, когда совокупность, из которой возникают данные, имеет распределение, которое является приблизительно «нормальным» (или гауссовым), тогда стандартное отклонение обеспечивает полезную основу для интерпретация данных с точки зрения вероятности.

    Нормальное распределение представлено семейством кривых, однозначно определяемых двумя параметрами: средним значением и стандартным отклонением генеральной совокупности.Кривые всегда имеют симметричную форму колокола, но степень сжатия или сплющивания колокола зависит от стандартного отклонения генеральной совокупности. Однако сам факт того, что кривая имеет форму колокола, не означает, что она представляет собой нормальное распределение, потому что другие распределения могут иметь подобную форму.

    Многие биологические характеристики соответствуют нормальному распределению достаточно близко для того, чтобы его можно было широко использовать — например, рост взрослых мужчин и женщин, артериальное давление в здоровой популяции, случайные ошибки во многих типах лабораторных измерений и биохимических данных.На рисунке 2.1 показана нормальная кривая, рассчитанная на основе диастолического артериального давления у 500 мужчин, среднего 82 мм рт. Ст., Стандартного отклонения 10 мм рт. Ст. Отмечены диапазоны, представляющие [+ -1SD, + 12SD и + -3SD] относительно среднего значения. Более обширный набор значений приведен в Таблице А печатного издания.

    Рисунок 2.1

    Причина, по которой стандартное отклонение является таким полезным показателем разброса наблюдений, заключается в следующем: если наблюдения следуют нормальному распределению, диапазон, охватываемый одним стандартным отклонением выше среднего и одним стандартным отклонением ниже него.

    включает около 68% наблюдений; диапазон двух стандартных отклонений выше и двух ниже () около 95% наблюдений; и трех стандартных отклонений выше и трех ниже () около 99.7% наблюдений. Следовательно, если мы знаем среднее значение и стандартное отклонение набора наблюдений, мы можем получить некоторую полезную информацию с помощью простой арифметики. Поместив одно, два или три стандартных отклонения выше и ниже среднего, мы можем оценить диапазоны, которые, как ожидается, будут включать около 68%, 95% и 99,7% наблюдений.

    Стандартное отклонение от разгруппированных данных

    Стандартное отклонение — это суммарная мера отличий каждого наблюдения от среднего.Если сложить сами различия, то положительный результат точно уравновесит отрицательный, и их сумма будет равна нулю. Следовательно, квадраты разностей складываются. Затем сумма квадратов делится на количество наблюдений за вычетом одного, чтобы получить среднее значение квадратов, и извлекается квадратный корень, чтобы вернуть измерения к единицам, с которых мы начали. (Деление на количество наблюдений минус один вместо самого числа наблюдений для получения среднего квадрата связано с тем, что необходимо использовать «степени свободы».В этих условиях их на единицу меньше, чем общее количество. Теоретическое обоснование этого не должно беспокоить пользователя на практике.)

    Чтобы получить интуитивное представление о степенях свободы, подумайте о выборе шоколада из коробки из n конфет. Каждый раз, когда мы выбираем шоколад
    , у нас есть выбор, пока мы не дойдем до последнего (обычно с орехом в нем!), И тогда у нас нет выбора. Таким образом, у нас есть n-1 выбор или «степень свободы».

    Расчет дисперсии показан в таблице 2.1 с 15 показаниями в предварительном исследовании концентрации свинца в моче (Таблица 1.2). Показания указаны в столбце (1). В столбце (2) записывается разница между каждым показанием и средним значением. Сумма разностей равна 0. В столбце (3) разности возведены в квадрат, а сумма этих квадратов приведена в нижней части столбца.

    Таблица 2.1

    Сумма квадратов разностей (или отклонений) от среднего, 9,96, теперь делится на общее количество наблюдений минус один, чтобы получить дисперсию.Таким образом,

    В этом случае мы находим:

    Наконец, квадратный корень из дисперсии дает стандартное отклонение:
    , из которого мы получаем
    .
    Эта процедура иллюстрирует структуру стандартного отклонения, в частности, то, что два крайних значения 0,1 и 3,2 вносят наибольший вклад в сумму квадратов разностей.

    Процедура калькулятора

    Большинство недорогих калькуляторов имеют процедуры, которые позволяют вычислять среднее и стандартное отклонения напрямую, используя режим «SD». Например, на современных калькуляторах Casio нажимаются клавиши SHIFT и ‘.’, И на дисплее должен появиться маленький символ« SD ». На более ранних Casios нужно нажимать INV и MODE, тогда как на Sharp 2nd F и Stat следует использовать. Данные сохраняются с помощью кнопки M +. Таким образом, установив калькулятор в режим «SD» или «Stat», из таблицы 2.1 мы вводим 0,1 M +, 0,4 M + и т. Д. Когда все данные введены, мы можем проверить правильность количества наблюдений, включенных Должны отображаться Shift и n, и «15». Среднее значение отображается как Shift и, а стандартное отклонение — как Shift и.Избегайте нажатия Shift и AC между этими операциями, так как это очищает статистическую память. На многих калькуляторах есть еще одна кнопка. В этом случае при вычислении стандартного отклонения используется делитель n, а не n — 1. На калькуляторе Sharp обозначается, тогда как обозначается s. Это значения «генеральной совокупности», и они получены при условии, что доступна вся генеральная совокупность или что интерес сосредоточен исключительно на имеющихся данных, и результаты не будут обобщаться (см. Главу
    3 для получения подробной информации о выборках и популяциях). .Поскольку такая ситуация возникает очень редко, ее следует использовать и игнорировать, хотя даже для средних размеров выборки разница будет небольшой. Не забудьте вернуться в нормальный режим перед возобновлением вычислений, потому что многие из обычных функций недоступны в режиме «Stat». На современных Casio это Shift 0. На более ранних Casios и Sharps повторяется последовательность, вызывающая режим «Stat». Некоторые калькуляторы остаются в «Стат»
    .
    даже в выключенном состоянии. Mullee (1) дает советы по выбору и использованию калькулятора.В формулах калькулятора используется соотношение «Правое выражение можно легко запомнить, используя выражение« среднее квадратов минус средний квадрат »». Выборочная дисперсия получается из приведенного выше уравнения, как видно из таблицы 2.1, где сумма квадратов наблюдений равна 43,7l.

    Таким образом, мы получаем

    то же самое значение, которое указано для итога в столбце (3). Следует проявлять осторожность, потому что эта формула включает вычитание двух больших чисел, чтобы получить маленькое, и может привести к неверным результатам, если числа очень большие.Например, попробуйте найти на калькуляторе стандартное отклонение 100001, 100002, 100003. Правильный ответ — 1, но многие калькуляторы выдадут 0 из-за ошибки округления. Решение состоит в том, чтобы вычесть большое число из каждого из наблюдений (скажем, 100000) и вычислить стандартное отклонение для остатков, а именно 1, 2 и 3.

    Стандартное отклонение от сгруппированных данных

    Мы также можем вычислить стандартное отклонение для дискретные количественные переменные. Например, помимо изучения концентрации свинца в моче 140 детей, педиатр спросил, как часто каждый из них осматривался врачом в течение года.После сбора информации он свел данные, представленные в столбцах (1) и (2) таблицы 2.2. Среднее значение рассчитывается путем умножения столбца (1) на столбец (2), добавления произведений и деления на общее количество наблюдений. Таблица 2.2.

    Как и для непрерывных данных, для вычисления стандартного отклонения мы возводим квадраты каждого из наблюдений по очереди. В этом случае наблюдение — это количество посещений, но поскольку у нас есть несколько детей в каждом классе, как показано в столбце (2), каждое число в квадрате (столбец (4)) необходимо умножить на количество детей.Сумма квадратов приведена в нижней части столбца (5), а именно 1697. Затем мы используем формулу калькулятора, чтобы найти дисперсию: и. Обратите внимание, что, хотя количество посещений обычно не распределяется, распределение достаточно симметрично относительно иметь в виду. Приблизительный диапазон 95% представлен в: Исключая двух детей без посещений и
    детей.
    шесть детей с шестью и более посещениями. Таким образом, восемь из 140 = 5,7% выходят за пределы теоретического 95% диапазона. Обратите внимание, что дискретные количественные переменные обычно имеют так называемое асимметричное распределение, то есть они не симметричны.Одним из ключей к отсутствию симметрии на основе производных статистических данных является существенное различие среднего и медианного значений. Другой случай — стандартное отклонение того же порядка, что и среднее значение, но результаты наблюдений должны быть неотрицательными. Иногда трансформация будет
    преобразовать искаженное распределение в симметричное. Когда данные подсчитываются, например количество посещений врача, часто помогает преобразование квадратного корня, а если нет нулевых или отрицательных значений, логарифмическое преобразование сделает распределение более симметричным.

    Преобразование данных

    Анестезиолог измеряет боль во время процедуры с помощью визуальной аналоговой шкалы 100 мм у семи пациентов. Результаты представлены в Таблице 2.3 вместе с преобразованием журнала (кнопка ln на калькуляторе). Таблица 2.3.

    Данные представлены на Рисунке 2.2, который показывает, что выбросы не выглядят столь значительными в зарегистрированных данных. Среднее значение и медиана составляют 10,29 и 2 соответственно для исходных данных со стандартным отклонением 20,22. Если среднее значение больше медианы, распределение имеет положительный перекос.Для зарегистрированных данных среднее значение и медиана составляют 1,24 и 1,10 соответственно, что указывает на более симметричное распределение зарегистрированных данных. Таким образом, было бы лучше проанализировать записанные преобразованные данные
    в статистических тестах, чем с использованием исходной шкалы. Рисунок 2.2 При представлении этих результатов будет дана медиана исходных данных, но следует пояснить, что статистический тест проводился на преобразованных данных. Обратите внимание, что медиана зарегистрированных данных совпадает с логарифмом медианы необработанных данных, однако это неверно для среднего значения.Среднее значение зарегистрированных данных не обязательно равно значению среднего значения необработанных данных.
    Антилогарифм (exp или на калькуляторе) среднего зарегистрированных данных известен как среднее геометрическое и часто равен
    .
    лучшая суммарная статистика, чем среднее значение для данных из положительно искаженных распределений. Для этих данных среднее геометрическое в 3,45 мм.

    Стандартное отклонение между субъектами и внутри субъектов

    Если проводятся повторные измерения, скажем, артериального давления у человека, эти измерения могут отличаться.Это внутри предметной или внутрипредметной изменчивости, и мы можем вычислить стандартное отклонение этих наблюдений. Если наблюдения близки по времени, это стандартное отклонение часто называют ошибкой измерения. Измерения, сделанные на разных объектах, различаются в зависимости от предметной или межпредметной вариабельности. Если бы было сделано много наблюдений над каждым человеком и было взято среднее значение, то можно было бы предположить, что внутрипредметная изменчивость была усреднена, а вариация средних значений обусловлена ​​исключительно межпредметной изменчивостью.Единичные наблюдения за людьми явно содержат смесь межпредметных и внутрипредметных вариаций. Коэффициент вариации (CV%) — это стандартное отклонение внутри объекта, деленное на среднее значение, выраженное в процентах. Его часто называют мерой повторяемости биохимических анализов, когда анализ проводится несколько раз на одном и том же образце. У него есть то преимущество, что он не зависит от единиц измерения, но также и многочисленные теоретические недостатки. Обычно бессмысленно использовать коэффициент вариации как меру вариативности между предметами.

    Общие вопросы

    Когда мне следует использовать среднее, а когда использовать медианное значение для описания моих данных
    ?

    Распространено заблуждение, что для нормально распределенных данных используется среднее значение, а для ненормально распределенных данных используется медиана. Увы, это не так: если данные распределены нормально, среднее значение и медиана будут близкими; если данные распределены ненормально, то полезную информацию могут дать как среднее, так и медианное значение. Рассмотрим переменную, которая принимает значение 1 для мужчин и 0 для женщин.Ясно, что это не распространяется нормально. Однако среднее значение дает долю мужчин в группе, тогда как среднее значение просто говорит нам, в какой группе было более 50% людей. Точно так же среднее значение упорядоченных категориальных переменных может быть более полезным, чем медиана, если упорядоченным категориям можно дать значимые оценки. Например, лекция может быть оценена от 1 (плохо) до 5 (отлично). Обычная статистика для обобщения результата — это среднее значение. В ситуации, когда небольшая группа находится на одном конце распределения (например, годовой доход), тогда медиана будет более «репрезентативной» для распределения.Мои данные должны иметь значения больше нуля, но при этом среднее и стандартное отклонение примерно одинакового размера. Как это произошло? Если данные имеют очень асимметричное распределение, то стандартное отклонение будет сильно завышено и не будет хорошей мерой изменчивости для использования. Как мы показали, иногда преобразование данных, такое как преобразование журнала, делает распределение более симметричным. Или укажите межквартильный размах.

    Список литературы

    1. Mullee M A.Как выбрать и пользоваться калькулятором. В: Как это сделать 2. Издательская группа BMJ
    , 1995: 58-62.

    Упражнения

    Упражнения 2.1

    В ходе кампании против оспы врач спросил, сколько раз 150 человек в возрасте 16 лет и старше были вакцинированы в эфиопской деревне. Получил следующие цифры: никогда — 12 человек; один раз, 24; дважды, 42; трижды, 38; четыре раза, 30; пять раз, 4. Каково среднее количество вакцинаций этим людям и каково стандартное отклонение? Ответ

    Упражнение 2.2

    Получите среднее и стандартное отклонение данных в и приблизительное значение
    95% диапазона. Ответ

    Упражнение 2.3

    Какие точки исключаются из среднего диапазона — 2SD на среднее + 2SD? Что
    доля данных исключена? Ответы
    Глава 2 Q3.pdf Ответ

    Стандартное отклонение | Как и когда использовать стандартное отклонение выборки и совокупности — мера разброса

    стандартное отклонение | Как и когда использовать стандартное отклонение выборки и совокупности — мера разброса | Статистика Лаэрд

    Введение

    Стандартное отклонение — это мера разброса оценок в наборе данных.Обычно нас интересует стандартное отклонение генеральной совокупности. Однако, поскольку нам часто предоставляются данные только из выборки, мы можем оценить стандартное отклонение генеральной совокупности по стандартному отклонению выборки. Эти два стандартных отклонения — стандартные отклонения выборки и совокупности — рассчитываются по-разному. В статистике нам обычно приходится рассчитывать стандартные отклонения выборки, поэтому именно на этом и будет сосредоточено внимание данной статьи, хотя также будет показана формула для стандартного отклонения генеральной совокупности.

    Когда использовать стандартное отклонение выборки или генеральной совокупности

    Обычно нам интересно знать стандартное отклонение генеральной совокупности, потому что наша совокупность содержит все интересующие нас значения. Следовательно, вы обычно рассчитываете стандартное отклонение генеральной совокупности, если: (1) у вас есть вся совокупность или (2) у вас есть выборка из более широкой совокупности, но вас интересует только эта выборка и вы не хотите обобщать свои выводы на совокупность.Однако в статистике нам обычно предоставляется выборка, по которой мы хотим оценить (обобщить) популяцию, и стандартное отклонение не является исключением. Следовательно, если все, что у вас есть, — это выборка, но вы хотите сделать заявление о стандартном отклонении генеральной совокупности, из которого взята выборка, вам необходимо использовать стандартное отклонение выборки. Часто может возникнуть путаница относительно того, какое стандартное отклонение использовать из-за того, что название «стандартное отклонение выборки» неправильно интерпретируется как означающее стандартное отклонение самой выборки, а не оценку стандартного отклонения генеральной совокупности, основанную на выборке.

    Какой тип данных следует использовать при вычислении стандартного отклонения?

    Стандартное отклонение используется вместе со средним значением для суммирования непрерывных данных, а не категориальных данных. Кроме того, стандартное отклонение, как и среднее значение, обычно подходит только в том случае, если непрерывные данные не сильно искажены или имеют выбросы.

    Примеры использования стандартного отклонения выборки или генеральной совокупности

    Q.Учитель ставит ученикам экзамен. Учитель хочет суммировать результаты, полученные учениками, в виде среднего значения и стандартного отклонения. Какое стандартное отклонение следует использовать?

    A. Стандартное отклонение численности населения. Почему? Потому что учителя интересуют только оценки этого класса учеников и никто другой.

    В. Исследователь набрал мужчин в возрасте от 45 до 65 лет для исследования физических упражнений, чтобы изучить маркеры риска сердечных заболеваний (например, холестерин). Какое стандартное отклонение, скорее всего, будет использовано?

    А.Стандартное отклонение выборки. Хотя это прямо не указано, исследователь, исследующий вопросы, связанные со здоровьем, не будет интересоваться только участниками своего исследования; они захотят показать, как результаты их выборки могут быть обобщены на все население (в данном случае на мужчин в возрасте от 45 до 65 лет). Следовательно, использование стандартного отклонения выборки.

    В. Один из вопросов национального консенсусного исследования касается возраста респондентов. Какое стандартное отклонение было бы использовано для описания разброса во всех возрастных группах, полученного в результате консенсуса?

    А.Стандартное отклонение населения. Национальный консенсус используется для получения информации о гражданах страны. По определению, это все население. Следовательно, будет использоваться стандартное отклонение совокупности.

    Какие формулы для стандартного отклонения?

    Формула стандартного отклонения для выборки :

    где,

    s = стандартное отклонение выборки
    = сумма …
    = выборочное среднее
    n = количество баллов в выборке.

    Стандартное отклонение населения Формула :

    где,

    = стандартное отклонение совокупности
    = сумма …
    = среднее значение совокупности
    n = количество баллов в выборке.

    Есть ли простой способ рассчитать стандартное отклонение?

    Да, у нас есть калькулятор стандартного отклонения выборки и генеральной совокупности, который также показывает, как все работает! Его можно найти в нашем разделе калькуляторов.

    Главная О нас Связаться с нами Положения и условия Конфиденциальность и файлы cookie © Lund Research Ltd, 2018 г. Определение стандартного отклонения

    Что такое стандартное отклонение?

    Стандартное отклонение — это статистика, которая измеряет дисперсию набора данных относительно его среднего значения и рассчитывается как квадратный корень из дисперсии. Стандартное отклонение рассчитывается как квадратный корень из дисперсии путем определения отклонения каждой точки данных относительно среднего значения. Если точки данных находятся дальше от среднего, в наборе данных имеется большее отклонение; таким образом, чем шире разброс данных, тем выше стандартное отклонение.

    Ключевые выводы:

    • Стандартное отклонение измеряет разброс набора данных относительно его среднего значения.
    • Волатильные акции имеют высокое стандартное отклонение, в то время как отклонение стабильных голубых фишек обычно довольно низкое.
    • С другой стороны, стандартное отклонение рассчитывает всю неопределенность как риск, даже если она в пользу инвестора, например, доходность выше среднего. {th} \ text {в наборе данных} \\ & \ overline {x} = \ text {Среднее значение набора данных} \\ & n = \ text {Количество точек данных в наборе данных} \ end {выровнено}
      Стандартное отклонение = n − 1∑i = 1n (xi −x) 2, где: xi = значение i-й точки в наборе данных x = среднее значение набора данных

      Расчет стандартного отклонения

      Стандартное отклонение рассчитывается следующим образом:

      1. Среднее значение вычисляется путем сложения всех точек данных и деления на количество точек данных.
      2. Дисперсия для каждой точки данных рассчитывается путем вычитания среднего значения из значения точки данных. Затем каждое из этих результирующих значений возводится в квадрат и суммируются результаты. Затем результат делится на количество точек данных за вычетом единицы.
      3. Квадратный корень из дисперсии — результат № 2 — затем используется для определения стандартного отклонения.

      Использование стандартного отклонения

      Стандартное отклонение — особенно полезный инструмент в инвестиционных и торговых стратегиях, поскольку он помогает измерять волатильность рынка и ценных бумаг, а также прогнозировать тенденции производительности.Например, что касается инвестирования, индексный фонд, вероятно, будет иметь низкое стандартное отклонение по сравнению с его эталонным индексом, поскольку цель фонда — воспроизвести индекс.

      С другой стороны, можно ожидать, что фонды агрессивного роста будут иметь высокое стандартное отклонение от относительных фондовых индексов, поскольку их управляющие портфелями делают агрессивные ставки для получения прибыли выше среднего.

      Более низкое стандартное отклонение не обязательно является предпочтительным. Все зависит от вложений и готовности инвестора взять на себя риск.Имея дело с величиной отклонений в своих портфелях, инвесторы должны учитывать свою терпимость к волатильности и свои общие инвестиционные цели. Более агрессивные инвесторы могут быть довольны инвестиционной стратегией, которая выбирает автомобили с волатильностью выше среднего, в то время как более консервативные инвесторы могут не делать этого.

      Стандартное отклонение — один из ключевых фундаментальных показателей риска, который используют аналитики, управляющие портфелем, консультанты. Инвестиционные фирмы сообщают о стандартном отклонении своих паевых инвестиционных фондов и других продуктов.Большой разброс показывает, насколько доходность фонда отклоняется от ожидаемой нормальной доходности. Поскольку эта статистика проста для понимания, она регулярно предоставляется конечным клиентам и инвесторам.

      Стандартное отклонение по сравнению с отклонением

      Дисперсия получается путем взятия среднего значения точек данных, вычитания среднего значения из каждой точки данных в отдельности, возведения в квадрат каждого из этих результатов, а затем взятия другого среднего значения этих квадратов. Стандартное отклонение — это квадратный корень из дисперсии.

      Дисперсия помогает определить размер разброса данных по сравнению со средним значением. По мере того, как дисперсия становится больше, происходит больше вариаций в значениях данных, и может быть больший разрыв между одним значением данных и другим. Если все значения данных близки друг к другу, дисперсия будет меньше. Однако это сложнее понять, чем стандартное отклонение, поскольку дисперсия представляет собой результат в квадрате, который не может быть значимо выражен на том же графике, что и исходный набор данных.

      Стандартные отклонения обычно легче изобразить и применить. Стандартное отклонение выражается в той же единице измерения, что и данные, что не обязательно относится к дисперсии. Используя стандартное отклонение, статистики могут определить, имеют ли данные нормальную кривую или другую математическую зависимость. Если данные ведут себя по нормальной кривой, то 68% точек данных будут находиться в пределах одного стандартного отклонения от среднего или среднего значения точки данных. Большие отклонения приводят к тому, что большее количество точек данных выходит за пределы стандартного отклонения.Меньшие отклонения приводят к большему количеству данных, близких к среднему.

      Большой недостаток

      Самый большой недостаток использования стандартного отклонения заключается в том, что на него могут влиять выбросы и экстремальные значения. Стандартное отклонение предполагает нормальное распределение и рассчитывает всю неопределенность как риск, даже если она в пользу инвестора, например, доходность выше среднего.

      Пример стандартного отклонения

      Скажем, у нас есть точки данных 5, 7, 3 и 7, всего 22.Затем вы разделите 22 на количество точек данных, в данном случае на четыре, что даст среднее значение 5,5. Это приводит к следующим определениям: x̄ = 5.5 и N = 4.

      Дисперсия определяется путем вычитания среднего значения из каждой точки данных, что дает -0,5, 1,5, -2,5 и 1,5. Затем каждое из этих значений возводится в квадрат, в результате получается 0,25, 2,25, 6,25 и 2,25. Значения квадратов затем складываются, в результате получается 11, которые затем делятся на значение N минус 1, что составляет 3, в результате получается дисперсия примерно 3.67.

      Затем вычисляется квадратный корень из дисперсии, что дает стандартное отклонение примерно 1,915.

      Или рассмотрим акции Apple (AAPL) за последние пять лет. Доходность акций Apple составила 12,49% в 2016 году, 48,45% в 2017 году, -5,39% в 2018 году, 88,98% в 2019 году и по состоянию на сентябрь 60,91% в 2020 году. 36,88%.

      Таким образом, абсолютное значение годовой прибыли за вычетом среднего составляет 24.39%, 11,57%, 42,27%, 52,1% и 24,03% соответственно. Затем все эти значения возводятся в квадрат, чтобы получить 0,059, 0,013, 0,179, 0,271 и 0,058. Выборочная дисперсия — это среднее значение квадрата разности, или 0,145, где значения в квадрате складываются и делятся на 4 (N минус 1). Квадратный корень из дисперсии используется для получения стандартного отклонения 38,08%.

      Калькулятор стандартного отклонения

      Укажите числа, разделенные запятыми, для расчета стандартного отклонения, дисперсии, среднего, суммы и погрешности.

      Калькулятор связанной вероятности | Калькулятор объема выборки | Статистический калькулятор

      Стандартное отклонение в статистике, обычно обозначаемое σ , является мерой вариации или дисперсии (относится к степени растяжения или сжатия распределения) между значениями в наборе данных. Чем ниже стандартное отклонение, тем ближе точки данных к среднему (или ожидаемому значению), μ . И наоборот, более высокое стандартное отклонение указывает на более широкий диапазон значений.Подобно другим математическим и статистическим концепциям, существует множество различных ситуаций, в которых может использоваться стандартное отклонение, и, следовательно, множество различных уравнений. Помимо выражения изменчивости популяции, стандартное отклонение также часто используется для измерения статистических результатов, таких как предел погрешности. При таком использовании стандартное отклонение часто называют стандартной ошибкой среднего или стандартной ошибкой оценки относительно среднего. Приведенный выше калькулятор вычисляет стандартное отклонение генеральной совокупности и стандартное отклонение выборки, а также приближения доверительного интервала.

      Стандартное отклонение населения

      Стандартное отклонение совокупности, стандартное определение σ , используется, когда можно измерить всю совокупность, и представляет собой квадратный корень из дисперсии данного набора данных. В случаях, когда выборка может быть произведена по каждому члену генеральной совокупности, можно использовать следующее уравнение, чтобы найти стандартное отклонение для всей генеральной совокупности:

      Где

      x i — индивидуальное значение
      μ — среднее / ожидаемое значение
      N — общее количество значений

      Для тех, кто не знаком с нотацией суммирования, приведенное выше уравнение может показаться сложным, но при обращении к его отдельным компонентам это суммирование не представляет особой сложности. i = 1 в суммировании указывает начальный индекс, т.е. для набора данных 1, 3, 4, 7, 8, i = 1 будет 1, i = 2 будет 3 и т. Д. . Следовательно, обозначение суммирования просто означает выполнение операции (x i — μ 2 ) для каждого значения до N , которое в данном случае равно 5, поскольку в этом наборе данных 5 значений.

      Пример: μ = (1 + 3 + 4 + 7 + 8) / 5 = 4,6
      σ = √ [(1 — 4.6) 2 + (3 — 4,6) 2 + … + (8 — 4,6) 2 )] / 5
      σ = √ (12,96 + 2,56 + 0,36 + 5,76 + 11,56) / 5 = 2,577

      Стандартное отклонение выборки

      Во многих случаях невозможно произвести выборку каждого члена в совокупности, что требует изменения приведенного выше уравнения, чтобы можно было измерить стандартное отклонение с помощью случайной выборки изучаемой совокупности. Обычным оценщиком для σ является стандартное отклонение выборки, обычно обозначаемое s .Стоит отметить, что существует множество различных уравнений для расчета стандартного отклонения выборки, поскольку, в отличие от выборочного среднего, стандартное отклонение выборки не имеет единой оценки, которая была бы беспристрастной, эффективной и имела бы максимальную вероятность. Приведенное ниже уравнение представляет собой «скорректированное стандартное отклонение выборки». Это скорректированная версия уравнения, полученная в результате модификации уравнения стандартного отклонения генеральной совокупности с использованием размера выборки в качестве размера генеральной совокупности, что устраняет часть систематической ошибки в уравнении.Однако объективная оценка стандартного отклонения очень сложна и варьируется в зависимости от распределения. Таким образом, «скорректированное стандартное отклонение выборки» является наиболее часто используемым средством оценки стандартного отклонения генеральной совокупности и обычно называется просто «стандартным отклонением выборки». Это гораздо лучшая оценка, чем его нескорректированная версия, но все же имеет значительную систематическую ошибку для небольших размеров выборки (N

      Где

      x i — одно значение выборки
      — среднее значение выборки
      N — размер выборки

      Пример работы с суммированием см. В разделе «Стандартное отклонение совокупности».Уравнение по существу такое же, за исключением члена N-1 в уравнении откорректированного отклонения выборки и использования значений выборки.

      Применение стандартного отклонения

      Стандартное отклонение широко используется в экспериментальных и промышленных условиях для проверки моделей на реальных данных. Примером этого в промышленных приложениях является контроль качества некоторых продуктов. Стандартное отклонение можно использовать для расчета минимального и максимального значения, в пределах которого какой-либо аспект продукта должен попадать в некоторый высокий процент времени.В случаях, когда значения выходят за пределы расчетного диапазона, может потребоваться внести изменения в производственный процесс для обеспечения контроля качества.

      Стандартное отклонение также используется в погодных условиях для определения различий в региональном климате. Представьте себе два города, один на побережье и один в глубине суши, с одинаковой средней температурой 75 ° F. Хотя это может вызвать убеждение в том, что температуры в этих двух городах практически одинаковы, реальность могла бы быть замаскирована, если бы учитывались только средние значения и игнорировалось стандартное отклонение.Прибрежные города, как правило, имеют гораздо более стабильные температуры из-за регулирования со стороны больших водоемов, поскольку вода имеет более высокую теплоемкость, чем земля; по сути, это делает воду гораздо менее восприимчивой к изменениям температуры, и прибрежные районы остаются более теплыми зимой и более прохладными летом из-за количества энергии, необходимого для изменения температуры воды. Следовательно, в то время как в прибрежном городе может быть диапазон температур от 60 ° F до 85 ° F в течение определенного периода времени, что приводит к среднему значению 75 ° F, во внутреннем городе может быть температура в диапазоне от 30 ° F до 110 ° F до результат то же среднее.

      Другой областью, в которой широко используется стандартное отклонение, является финансы, где оно часто используется для измерения риска, связанного с колебаниями цен на некоторые активы или портфели активов. Использование стандартного отклонения в этих случаях позволяет оценить неопределенность будущей прибыли от данной инвестиции. Например, при сравнении акции A, которая имеет среднюю доходность 7% со стандартным отклонением 10%, с акцией B, которая имеет такую ​​же среднюю доходность, но стандартное отклонение 50%, первая акция, несомненно, будет более безопасным вариантом. поскольку стандартное отклонение запаса B значительно больше, при точно такой же доходности.Это не означает, что в данном сценарии акции A определенно являются лучшим вариантом для инвестиций, поскольку стандартное отклонение может исказить среднее значение в любом направлении. В то время как акция A имеет более высокую вероятность средней доходности, близкой к 7%, акция B потенциально может обеспечить значительно больший доход (или убыток).

      Это лишь несколько примеров того, как можно использовать стандартное отклонение, но существует гораздо больше. Как правило, вычисление стандартного отклонения полезно в любое время, когда необходимо знать, насколько далеко от среднего может быть типичное значение из распределения.

      Стандартное отклонение | Пошаговое руководство с формулами

      Стандартное отклонение — это средняя величина изменчивости в вашем наборе данных. Он сообщает вам, в среднем, насколько далеко каждое значение от среднего.

      Высокое стандартное отклонение означает, что значения обычно далеки от среднего, в то время как низкое стандартное отклонение указывает, что значения сгруппированы близко к среднему.

      Что вам говорит стандартное отклонение?

      Стандартное отклонение — полезная мера разброса для нормального распределения .

      В нормальном распределении данные распределяются симметрично без перекоса. Большинство значений группируются вокруг центральной области, причем значения сужаются по мере удаления от центра. Стандартное отклонение показывает, насколько в среднем разбросаны ваши данные от центра распределения.

      Многие научные переменные подчиняются нормальному распределению, включая рост, результаты стандартизированных тестов или оценки удовлетворенности работой. Когда у вас есть стандартные отклонения различных выборок, вы можете сравнить их распределения с помощью статистических тестов, чтобы сделать выводы о более крупных популяциях, из которых они произошли.

      Пример: сравнение различных стандартных отклонений. Вы собираете данные об оценках удовлетворенности работой трех групп сотрудников, используя простую случайную выборку.

      Средние ( M ) оценки одинаковы для каждой группы — это значение на оси x, когда кривая находится на пике. Однако их стандартные отклонения ( SD ) отличаются друг от друга.

      Стандартное отклонение отражает дисперсию распределения. Кривая с наименьшим стандартным отклонением имеет высокий пик и небольшой разброс, тогда как кривая с наибольшим стандартным отклонением более плоская и широко распространенная.

      Эмпирическое правило

      Стандартное отклонение и среднее значение вместе могут сказать вам, где находится большинство значений в вашем распределении, если они соответствуют нормальному распределению.

      Эмпирическое правило , или правило 68-95-99.7, говорит вам, где находятся ваши ценности:

      • Около 68% оценок находятся в пределах 2 стандартных отклонений от среднего значения,
      • Около 95% оценок находятся в пределах 4 стандартных отклонений от среднего значения,
      • Около 99.7% оценок находятся в пределах 6 стандартных отклонений от среднего.

      Пример: стандартное отклонение в нормальном распределении Вы проводите тест на вспоминание памяти для группы студентов. Данные соответствуют нормальному распределению со средним баллом 50 и стандартным отклонением 10.

      Следуя эмпирическому правилу:

      • Около 68% оценок находятся в диапазоне от 40 до 60.
      • Около 95% оценок находятся в диапазоне от 30 до 70.
      • Около 99,7% оценок находятся в диапазоне от 20 до 80.

      Эмпирическое правило — это быстрый способ получить обзор ваших данных и проверить любые выбросы или экстремальные значения, которые не соответствуют этому шаблону.

      Для ненормальных распределений стандартное отклонение является менее надежным показателем изменчивости и должно использоваться в сочетании с другими показателями, такими как размах или межквартильный размах.

      Формулы стандартного отклонения для популяций и выборок

      Для расчета стандартных отклонений используются разные формулы в зависимости от того, есть ли у вас данные от всей генеральной совокупности или от выборки.

      Стандартное отклонение совокупности

      Когда вы собрали данные от каждого члена населения, который вас интересует, вы можете получить точное значение стандартного отклонения населения.

      Формула стандартного отклонения совокупности выглядит так:

      Формула Пояснение
      • σ = стандартное отклонение совокупности
      • ∑ = сумма…
      • X = каждое значение
      • μ = среднее значение по совокупности
      • N = количество значений в генеральной совокупности

      Стандартное отклонение выборки

      Когда вы собираете данные из выборки, стандартное отклонение выборки используется для оценок или выводов о стандартном отклонении генеральной совокупности.

      Формула стандартного отклонения выборки выглядит так:

      Формула Пояснение
      • с = стандартное отклонение выборки
      • ∑ = сумма…
      • X = каждое значение
      • x̅ = выборочное среднее
      • n = количество значений в выборке

      В случае выборок мы используем в формуле n — 1, потому что использование n даст нам смещенную оценку, которая постоянно недооценивает изменчивость.Стандартное отклонение выборки будет, как правило, ниже, чем реальное стандартное отклонение генеральной совокупности.

      Уменьшение выборки n до n — 1 делает стандартное отклонение искусственно большим, давая вам консервативную оценку изменчивости.

      Хотя это не беспристрастная оценка, это менее предвзятая оценка стандартного отклонения: лучше переоценить, чем недооценить изменчивость в выборках.

      Какая у вас оценка за плагиат?

      Сравните вашу статью с более чем 60 миллиардами веб-страниц и 30 миллионами публикаций.

      • Лучшая программа для проверки плагиата 2020 года
      • Отчет о плагиате и процентное содержание
      • Самая большая база данных о плагиате

      Scribbr Проверка на плагиат

      Шаги для расчета стандартного отклонения

      Стандартное отклонение обычно рассчитывается автоматически любым программным обеспечением, которое вы используете для статистического анализа. Но вы также можете рассчитать его вручную, чтобы лучше понять, как работает формула.

      Есть шесть основных шагов для определения стандартного отклонения вручную. Мы будем использовать небольшой набор данных из 6 баллов, чтобы пройти через шаги.

      Набор данных
      46 69 32 60 52 41

      Шаг 1 : Найдите среднее значение

      Чтобы найти среднее значение, сложите все баллы, затем разделите их на количество баллов.

      Среднее (x̅)
      x̅ = (46 + 69 + 32 + 60 + 52 + 41) ÷ 6 = 50

      Шаг 2 : Найдите отклонение каждой оценки от среднего

      Вычтите среднее значение из каждой оценки, чтобы получить отклонения от среднего.

      Поскольку x̅ = 50, здесь мы убираем 50 из каждого результата.

      Оценка Отклонение от среднего
      46 46-50 = -4
      69 69-50 = 19
      32 32-50 = -18
      60 60-50 = 10
      52 52-50 = 2
      41 41-50 = -9

      Шаг 3 : Возвести в квадрат каждое отклонение от среднего

      Умножьте каждое отклонение от среднего на само себя.Это приведет к положительным числам.

      Квадратные отклонения от среднего
      (-4) 2 = 4 × 4 = 16
      19 2 = 19 × 19 = 161
      (-18) 2 = -18 × -18 = 324
      10 2 = 10 × 10 = 100
      2 2 = 2 × 2 = 4
      (-9) 2 = -9 × -9 = 81

      Шаг 4 : Найдите сумму квадратов

      Сложите все квадраты отклонений.Это называется суммой квадратов.

      Сумма квадратов
      16 + 361 + 324 + 100 + 4 + 81 = 886

      Шаг 5: Найдите отклонение

      Разделите сумму квадратов на n — 1 (для выборки) или N (для генеральной совокупности) — это дисперсия.

      Поскольку мы работаем с размером выборки 6, мы будем использовать n — 1, где n = 6.

      Разница
      886 ÷ (6-1) = 886 ÷ 5 = 177,2

      Шаг 6 : Найдите квадратный корень из дисперсии

      Чтобы найти стандартное отклонение, мы извлекаем квадратный корень из дисперсии.

      Стандартное отклонение
      √177,2 = 13,31

      Исходя из того, что SD = 13.31, можно сказать, что каждая оценка отклоняется от среднего в среднем на 13,31 балла.

      Почему стандартное отклонение является полезным показателем изменчивости?

      Хотя существуют более простые способы расчета вариабельности, формула стандартного отклонения позволяет взвешивать неравномерно распределенные образцы больше, чем равномерно распределенные. Более высокое стандартное отклонение говорит о том, что распределение не только более рассредоточено, но и более неравномерно.

      Это означает, что он дает вам лучшее представление об изменчивости ваших данных, чем более простые меры, такие как среднее абсолютное отклонение (MAD).

      MAD похоже на стандартное отклонение, но его легче вычислить. Во-первых, вы выражаете каждое отклонение от среднего в абсолютных значениях, преобразуя их в положительные числа (например, -3 становится 3). Затем вы вычисляете среднее значение этих абсолютных отклонений.

      В отличие от стандартного отклонения, вам не нужно вычислять квадраты или квадратные корни чисел для MAD. Однако по этой причине он дает менее точную меру изменчивости.

      Давайте возьмем две выборки с одной и той же центральной тенденцией, но с разной степенью изменчивости.Образец B более изменчив, чем образец A.

      Значения Среднее значение Среднее абсолютное отклонение Стандартное отклонение
      Образец A 66, 30, 40, 64 50 15 17,8
      Образец B 51, 21, 79, 49 50 15 23,7

      Для выборок с одинаковыми средними отклонениями от среднего MAD не может различать уровни разброса.Стандартное отклонение более точное: оно выше для выборки с большим разбросом отклонений от среднего.

      Возведя в квадрат разности от среднего, стандартное отклонение более точно отражает неравномерную дисперсию. Эта ступень более взвешивает крайние отклонения, чем небольшие отклонения.

      Однако это также делает стандартное отклонение чувствительным к выбросам.

      Часто задаваемые вопросы о стандартном отклонении

      Что вам говорит стандартное отклонение?

      Стандартное отклонение — это средняя величина изменчивости в вашем наборе данных.Он сообщает вам, в среднем, насколько далеко каждая оценка отличается от среднего значения.

      В нормальных распределениях высокое стандартное отклонение означает, что значения обычно далеки от среднего, в то время как низкое стандартное отклонение указывает, что значения сгруппированы близко к среднему.

      Что такое эмпирическое правило?

      Эмпирическое правило, или 68-95-99.Правило 7 говорит вам, где большинство значений лежит в нормальном распределении:

      • Около 68% значений находятся в пределах 1 стандартного отклонения от среднего.
      • Около 95% значений находятся в пределах 2 стандартных отклонений от среднего.
      • Около 99,7% значений находятся в пределах 3 стандартных отклонений от среднего.

Добавить комментарий

Ваш адрес email не будет опубликован. Обязательные поля помечены *